Download as pdf or txt
Download as pdf or txt
You are on page 1of 202

Test Information

Test Name ALL INDIA DAMS NEET-PG 2020 CBT-4 Total Questions 300

Test Type Examination Difficulty Level Difficult

Total 210
1200 Duration
Marks minutes

Test Question Language:-ENGLISH

1. A 40 yrs. old man had a stroke & developed Ipsilateral paralysis & atrophy of tongue, contralateral
loss of vibration sense, contralateral hemiplegia & contralateral Babinskis sign. Thrombosis of which
artery results in these neurologic Defect–

a. Anterior spinal

b. Posterior spinal

c. Posterior inferior cerebellar

d. Anterior inferior cerebellar.

Solution. A
Thrombosis of anterior spinal artery results in medial medullary syndrome. This includes
contralateral hemiparesis of trunk & extremities, contralateral loss of proprioception, tactile
localization & discrimination from trunk & extremities, & ipsilateral flaccid paralysis of tongue.

Answer. a
2. Identify the following slide-

a. Serous salivary gland

b. Mucus salivary gland

c. Mixed salivary gland

d. Pancreas

Solution. D
The exocrine part of pancreas represents a serous salivary gland. The endocrine part represents
Islets of Langerhans.

Answer. d

3. A 52 yrs. old man is brought to emergency dept. by a friend because of 5 day history of fever &
cough productive of purulent sputum. One week ago he was woken by episode of heavy coughing while
lying on back. He drinks large amount of alcohol & spends most of the time on bed since his wife
passed away 2 months ago. His temperature is 38*C, pulse is 96/min, RR is 24/min. A CT scan of chest
is mostly to show pulmonary infiltrate in which of following location -

a. Posterior basal segment of right lower lobe

b. Superior segment of right lower lobe

c. Posterior segment of right upper lobe

d. Posterior basal segment of left lower lobe

Solution. B
Part of lung affected in supine position is apical segment of Right lower lobe ( Right Superior
Segment). Foreign body entering the nose enters into posterior basal segment of right lower lobe.

Answer. b
4. A 21 yrs old man is brought to emergency department following an assault. He was attempting to
escape & turned away just as his attacker lunged towards him from behind with a knife. The patient
suffered a 4cm wide penetrating injury with significant associated blood loss. Following initial
stabilisation he is transferred to operating room. Surgical exploration reveals injury to a muscle that
assist internal rotation of shoulder & is innervated by thoracodorsal nerve. Which of the following
muscle as depicted in cadaveric specimen is most likely to be injured in this patient

a. A

b. B

c. C

d. D

Solution. B
The muscle is Latissimus dorsi. Causes adduction, extension & medial rotation at shoulder joint.
Supplied by thoracodorsal nerve / nerve to latissimus dorsi.
A – Trapezius
C – Deltoid
D – Triceps

Answer. b

5. A 62 yr old man comes to physician because of intermittent groin pain. He is a construction worker
& says that pain gets worse on lifting heavy objects & after a long day of working. Physical examination
shows a bulge above the inguinal ligament. The bulge increases in size when he bends down. A groin
USG reveals that the mass originates medial to inferior epigastric vessels. The patient’s condition is
most likely caused by which of the following –

a. Breakdown of fascia transversalis

b. Patent processes vaginalis

c. Weakness of rectus abdominis muscle

d. Widening of femoral ring.

Solution. A
A hernia above & medial to inguinal ligament is medial direct inguinal hernia. This is due to
defect in fascia transversalis. A hernia lateral to inferior epigastric vessels is indirect – due to
patent Processes vaginalis. Widening of femoral ring leads to Femoral hernia.

Answer. a
6. True regarding the following slide -

a. A - Branch of posterior cord

b. B – Lies lateral to brachial artery in cubital fossa

c. C – Is cutaneous in its supply.

d. D – Compression leads to Carpel Tunnel Syndrome.

Solution. C
A – musculocutaneos nerve – branch of lateral cord.
B – Median nerve – lies medial to brachial artery in cubital fossa.
C – Superficial branch of radial nerve – is cutaneous in supply.
D – Ulnar nerve – compression between the two heads of flexor carpi ulnaris leads to cubital
tunnel syndrome.

Answer. c
7. A 34 yrs. old man is brought to emergency department 3 hrs. after being bitten by a snake. He was
hiking in Arizona desert when he accidently stepped on the snake & it bit in his right leg. His pulse is
135/min * BP is 104/81mm of Hg. Examination of right lower leg shows swelling & blistering. Right
ankle dorsiflexion elicits severe pain. In addition to administering antivenom patient undergoes
fasciotomy. Two weeks later he reports difficulty in walking. Neurologic examination shows loss of
sensation over lower part of lateral side of right leg& dorsum of right foot. Right foot eversion is 1/5.
There is no weakness in dorsiflexion.. Which of the following nerves is most likely to be injured in the
patient –

a. Sural nerve

b. Deep peroneal nerve

c. Superficial peroneal nerve

d. Saphaneous nerve

Solution. C
The superficial peroneal is musculocutaneous nerve of lower limb. It supplies peroneus longus &
Peroneus brevis ( causes eversion of foot ) & skin over lateral aspect of leg, dorsum of foot & all
web spaces except first web space.

Answer. c
8. In the slide --- One of the following Representation is false -

a. A – Falciform ligament

b. B - Lesser omentum

c. C – Greater omentum

d. D – Lienorenal ligament

Solution. C
The following slides shows derivatives of dorsal & ventral mesogastrium. The liver develops in
ventral mesogastrium - splits it into – A - Falciform ligament, B – Lesser omentum. The spleen
grows in dorsal mesogastrium. Divides it into - C – gastrosplenic & D – lienorenal ligament.

Answer. c
9. A 64 yrs. old man comes to physician because of two week H/O intractable hiccups & shortness of
breath on exertion. He also has a 1 month H/O left shoulder pain. He has smoked one packet of
cigarette daily for 35 yrs. Physical examination shows decreased breath sounds at the left lung base.
An x-ray of chest shows 3cm peri-hilar mass & elevation of left hemi-diaphragm. The patients symptoms
are most likely caused by injury to a nerve that also innervates which of the following structure –

a. Visceral pleura

b. Fibrous pericardium

c. Ciliary muscle

d. Serratus anterior muscle

Solution. B
The nerve involved is phrenic nerve. It supplies diaphragm ( motor) & diaphragmatic &
mediastinal pleura. The fibrous pericardium & parietal layer of serous pericardium. The visceral
pleura & visceral layer of serous pericardium are supplied by Autonomic nerves. Serratus anterior
is supplied by long thoracic nerve. Sphincter pupillae & ciliaris muscle are supplied by
occulomotor nerve.

Answer. b
10. One of the following statement is False-

a. Lies medial in the floor of Lateral ventricle

b. Is a part of Diencephalon

c. Represents the Callosal sulcus

d. Is known as Cuneus.

Solution. A
A – Caudate nucleus. In the floor of lateral ventricle lies thalamus ( medial ) & caudate nucleus (
lateral).
B - Thalamus -- is a part of diencephalon.
C – Callosal sulcus – the ACA lies in it.
D – Cuneus – the part of brain lying between parieto-occipital & calcarine sulcus is called Cuneus.

Answer. a

11. A 32 yrs. old man comes to clinic of 2 day H/O tingling sensation in the right forearm. He reports
that his symptoms started after he lifted heavy weights in the gym. Physical Examination shows loss of
sensation on lateral aspect of right forearm. Sensation over thumb is intact. Range of motion of neck is
normal. Further examination of patient is most likely to show weakness of which of the following
actions –

a. Shoulder abduction

b. Elbow flexion

c. Forearm pronation

d. Wrist extension

Solution. B
The musculocutaneous nerve is involved. It supplies biceps, brachialis & coracobrachialis. It also
supplies skin over lateral aspect of forearm. Paralysis of brachialis leads to loss of flexion of
elbow.

Answer. b
12. One of the following statement is False-

a. A – supplied by Tibial nerve

b. B – Supplied by Superficial Peroneal nerve

c. C – Inserts on the base of 1st Metatarsal bone

d. D – Causes dorsiflexion of foot

Solution. B
A – Tendoachillis - -supplied by tibial nerve
B – Peroneus tertius – supplied by deep peroneal nerve
C – Peroneus longus – inserted on base of first metatarsal & medial cuneiform bone.
D – Extensor digitorum longus – causes dorsiflexion of foot.

Answer. b
13. Regarding the slide – One of the following statement is false -

a. A – Is related to Submandibular ganglion

b. B – Inserted into Pterygoid fovea of mandible

c. C – Lies in the Carotid Sheath

d. D - Carries secretomotor fibers to Parotid gland.

Solution. C
A – Lingual nerve – suspends the submandibular ganglion from it.
B – Lateral pterygoid – inserts on pterygoid fovea on neck of mandible
C – External carotid artery – lies outside the carotid sheath.
D – Auriculotemporal nerve – carries secretomotor fibers to parotid gland.

Answer. c
14. Regarding the slide - One of the statement is false -

a. A – Represented by septum Primum

b. B – AV node lies here

c. C – Is derived from left venous valve

d. D – Derived from primitive atrium.

Solution. C
A- Fossa ovalis -- represented by Septum primum
B- Triangle of Kochs. The AV node lies here
C- Eustachian valve – derived from right venous valve
D- Crista terminalis – derived from primitive atrial chamber.

Answer. c

15. A 44yr old man comes to office for evaluation of neck lump. He first noticed the lump 6 months ago
while he was shaving & says it seems to be growing in size. Physical examination reveals a nodular
mass in left thyroid lobe. He has no pain, shortness of breath, cough, hoarseness or difficulty in
swallowing. Serum TSH level is normal. Fine needle aspiration biopsy is consistent with papillary
thyroid cancer. A total thyroidectomy is performed. While attempting to ligate the inferior thyroid
artery the surgeon accidently damages the structure that is in close proximity to it. Which of the
following nerves was most likely injured –

a. Accessory

b. Recurrent laryngeal nerve

c. Ansa cervicalis

d. Hypoglossal nerve.

Solution. B
The inferior thyroid artery accompanies Recurrent laryngeal nerve. It supplies all the muscles of
larynx except cricothyroid- supplied by external laryngeal nerve.

Answer. b
16. Regarding the slide - One of the following statement is False -

a. A – Represents the artery of Hindgut.

b. B – Derived from Mesonephric duct

c. C – Related to Second part of duodenum

d. D – Is longer on the left side than right.

Solution. C
A – inferior mesenteric artery – is the artery of hindgut
B – Ureter – derived from ureteric bud – derived from mesonephric duct
C – IVC – relates to third part of duodenum
D – left renal vein - is longer on the left side.

Answer. c
17. A 53 yrs. old man is brought to emergency by his wife after a sudden onset of dizziness, nausea &
left sided weakness. His BP is 165/95mm hg. Neurological examination shows decreased muscle tone &
hype-reflexia in upper & lower extremities. His gait is unsteady & he falls to left while attempting to
walk. CT scan of head reveals a small lesion in left lateral pons that involves the nucleus of a cranial
nerve that exits from brain at middle cerebellar peduncle. Which additional findings is most likely in
this patient –

a. Difficulty in chewing & deviation of jaw to the left.

b. Impaired vision & absent direct pupillary light reflex of left eye

c. Deviation of tongue to left when protruded

d. Hoarseness & deviation of uvula to right

Solution. A
The nerve attached at junction of pons with middle cerebellar peduncle is trigeminal nerve (
motor & sensory root). The motor root joins mandibular nerve & supplies the muscles of first
pharyngeal arch. Paralysis of left lateral pterygoid – causes deviation of jaw to same side.
Lesion of 11th nerve causes deviation of protruded tongue.
Lesion of 10th nerve causes deviation of uvula to same side.
Pupillary light reflex – Afferent – optic nerve & Efferent – 3rd nerve.

Answer. a

18. Which of the following is present on the β subunit of the Na+-K+ ATPase pump?

a. Na+ binding site

b. Phosphorylation site

c. ATP binding site

d. Glycosylation site

Solution. D; Glycosylation site


Na+-K+ ATPase pump is a heterodimer.
It has two subunits- α subunit with a molecular weight of 100,000 and β subunit with molecular
weight of 55,000.
β subunit is a glycoprotein, has one membrane-spanning domain with three glycosylation sites.
α subunit probably spans the membrane 10 times.
α subunit has three intracellular binding sites- Na+ binding site, ATP binding site and
phosphorylation
site. It has two extracellular binding sites- K+ binding site and Ouabain binding site.
The movement of Na+ and K+ is a two-stage process. When Na+ binds to α subunit, ATP also
binds
and is converted to ADP by the ATPase enzyme, with the phosphate being transferred to Asp 376,
the phosphorylation site. This causes a change in the configuration of the protein, extruding Na+
into
the ECF. K+ then binds extracellularly, dephosphorylating the α subunit, which returns to its
previous
conformation returning K+ into the cytoplasm.

Answer. d
19. A 65-year-old avid male hiker, resident of Manali (at 5000 ft) was hiking in Sikkim with two
companions. They hiked to Nathula in Sikkim at 12,000 ft in one day. He suddenly complained of
extreme tiredness, light headedness, dizziness, difficulty in breathing. He began coughing and
complained of tightness in the chest. He was taken to the emergency of a local hospital. Examination
revealed respiratory rate of 32 per min, heart rate of 125/min, bp 180/100 mm Hg, bluish discoloration
of fingertips, crackles and wheeze on the right lower lobe. His X-ray chest was done and is shown:-

Which of the following is not a possible reason for the man’s condition?

a. Hypoxia induced vasoconstriction of pulmonary capillaries

b. Increased pressure in the pulmonary capillary beds

c. Disruption of the endothelial barrier resulting in leakage of fluid and proteins into the alveoli

d. Hypoxia causing vasodilation of pulmonary capillaries resulting in increased fluid formation

Solution. D; Hypoxia causing vasodilation of pulmonary capillaries resulting in increased fluid


formation The proposed mechanism for development of HAPE is uneven hypoxia-induced
vasoconstriction resulting in increased pressures in capillary beds, subsequent fluid shifts, and
vascular leakage. Disruption of the endothelial barrier results in fluid, proteins, and even
hemorrhage into alveolar spaces. On a molecular level, the proposed mechanisms for the
development of pulmonary edema at altitude include the nitric oxide pathway, the renin-
angiotensin system, heat shock protein, pulmonary surfactant proteins, and hypoxia inducible
factor.

Answer. d
20. A new seizure drug is being investigated that binds to and activates GABA-A receptors in the CNS.
During their experiments, a researcher uses a micro electrode to measure the RMP of a neuron. He
records RMP of -70mV. Which of the following membrane potentials is likely to be recorded following
exposure to the drug?

a. -75mV

b. -65mV

c. 0mV

d. +61mV

Solution. A; -75mV
GABA-A receptors are ionotropic receptors which cause opening of chloride channels, leading to
hyperpolarization of the cell. GABA-A agonist will, therefore, cause the RMP of the cell to become
more negative or -75mV.

Answer. a
21. Which of the following is greater in afterloaded muscle contractions as compared to preloaded
muscle contractions?

a. Latent period

b. Contraction period

c. Relaxation period

d. Work done

Solution. A; Latent period.


• Preload is the load placed on a muscle before the muscle contracts.
• It serves to stretch the muscle sarcomeres, thus producing a passive tension across the muscle.
• This passive tension increases muscle contraction in two ways:
(i) It adds an elastic recoil force to the muscle during its contraction;
(ii) It stretches the muscle to its resting length, producing the optimum length-tension
relationship for active force generation.
• In real life situations, it is a common practice to pre-stretch a muscle using antagonist muscles.
• After load is an opposing force which the muscle encounters immediately after it starts
contracting. It is a force that a muscle must overcome before an observable shortening of the
muscle can occur.
• The contraction and work done of an after-loaded muscle is comparatively lesser.

Answer. a
22. A 30-year old woman presents with progressive shortening of breath. Which of the following corresponds
with chronic blood loss?

a. A

b. B

c. C

d. D

Solution. A
Oxygen content = (oxygen in combination with Hb) + Dissolved oxygen
= [Hb (gm/dL) X 1.34 X % saturation] + [PO2 (mm Hg) X .003mL/dL/mm Hg]
Decrease in oxygen content of the blood is caused by:-
• Decrease in Hb (chronic blood loss, iron deficiency anemia, hemolytic anemia, etc.)
• Decrease in % saturation (CO poisoning, methemoglobinemia, sulphemoglobinemia)
• Decrease in PO2 (high altitude, hypoventilation due to any cause)

Answer. a
23. Which of the following is expected to decrease the measured airway resistance?

a. Stimulation of sympathetic nerves to airways

b. Adenosine

c. Release of histamine by mast cells

d. Cool air

Solution. A
Stimulation of sympathetic nerves causes bronchodilation. Adenosine, release of histamine and
cool air itself are bronchoconstrictors.

Answer. a

24. What is the lung volume at the end of maximum expiration?

a. ERV

b. Vital capacity

c. FRC

d. Residual volume

Solution. D; Residual volume


The lung volume at the end of maximum expiration is the residual volume. Residual volume is
1200 mL in males and 1100mL in females.
Residual volume, FRC and TLC cannot be measured by routine spirometery.

Answer. d

25. Which of the following tracts innervates the A gamma motor neurons?

a. Lateral corticospinal tract

b. Vestibulospinal tract

c. Reticulospinal tract

d. Rubrospinal tract

Solution. C
Pontine reticulospinal tract increases the A gamma motor neuron discharge and medullary
reticulospinal tract decreases the A gamma motor neuron discharge. All others supply the A alpha
motor neurons.

Answer. c
26. Phospholamban in cardiac and smooth muscle cells in which of the following?

a. Excitatory protein which increases SERCA activity

b. Inhibitory protein which decreases SERCA activity

c. Excitatory protein which increases Na+-Ca++ antiport

d. Inhibitory protein which decreases Na+-Ca++ antiport

Solution. B
Phospholamban is a 6kDa protein that tonically inhibits the SERCA pump by increasing its Km for
calcium. It is present in both cardiac and smooth muscle cells.
Phosphorylation of phospholamban relieves this inhibition, speeding up SERCA activity, therefore
lowering the cytoplasmic calcium. Phosphorylation of phospholamban is controlled by protein
kinases that are, in turn, controlled by agonists. Epinephrine binds to beta receptors on the
membrane surface, which are coupled to a heterotrimeric G- protein (Gs) tha activates adenyl
cyclase. This increases the intracellular c-AMP which in turn activates protein kinase (PKA).

Answer. b

27. Acidification of the urine occurs in which of the following segments of the nephron?

a. PCT

b. Loop of henle

c. DCT

d. CD

Solution. D
Though the maximum secretion of H+ occurs in the PCT, acidification of urine occurs in the late
DCT and CD.

Answer. d
28. Which of the following GI secretions has the highest pH?

a. Saliva

b. Succus entericus

c. Pancreatic juice

d. Brunners’ gland secretion

Solution. D
Saliva has a ph of 6.0 to 8.0 (usually 7.0 to 8.0).
Intestinal secretion or Succus entericus has apH of 7.0 to 8.0.
Pancreatic secretions have a pH of upto 8.8.
Brunners’ gland secretion has a pH of up to 9.3.

Answer. d

29. What is the duration of the nodal delay?

a. 0.06s

b. 0.07s

c. 0.08s

d. 0.09s

Solution. D
Duration of the nodal delay is 0.09s or 92millisec. The impulse slows down as it passes through
the AV node because (i) AV node has small diameter fibres which conduct at a velocity of
0.05m/sec. (ii) AVN has very few or no gap junctions.
Advantage of nodal delay is that the atria contract ahead of the ventricles.

Answer. d

30. Which of the following nerve fibres is the least susceptible to local anesthetics?

a. Type A

b. Type B

c. Type C

d. Type D

Solution. C
A gamma nerve fibres are the most susceptible to local anesthetics and C are the least
susceptible.
C fibres are least susceptible to pressure, hypoxia and local anesthetics.

Answer. c
31. Sodium iodide symporter is present in all of the following except

a. Thyroid cells

b. Intestinal epithelial cells

c. Pancreas

d. Gestational breast

Solution. C
NIS (secondary active co-transport) is present in
- Thyroid cells
- Salivary glands
- Stomach
- Intestinal epithelial cells (apical membrane)
- Gestational breast

Answer. c

32. Which of the following is due to the axon reflex?

a. Red reaction

b. Flare

c. Wheal

d. White reaction

Solution. B
When the skin is stroked firmly with appointed instrument, the triple response (also k/a Lewis’
triple response), which is a normal response to injury, is seen. It consists of the following:-
a. Red reaction (k/a red dermographism)- there is reddening at the site which appears in about 10
seconds. This initial redness is due to capillary dilatation, a direct response of the capillaries to
pressure.
b. Flare- this is a spreading redness from the site of injury due to arteriolar dilatation. This is due
to the “axon reflex”, a response in which impulses initiated in sensory nerves by the injury are
relayed antidromically down other branches of the sensory nerve fibers. The transmitter released
at the central termination of the sensory C fiber neurons is substance P. Substance P and CGRP
are present in all parts of neurons. Both dilate arterioles and, in addition, substance P causes
extravasation of fluid.
The axon reflex persists after total sympathectomy. Flare is absent in locally anesthetized skin
and in denervated skin after the sensory nerves have degenerated, but is present after a nerve
block or section above the site of injury. This indicates that flare is due to the axon reflex, a neural
response.
c. Wheal- swelling or local edema is due to increased permeability of the capillaries and post
capillary venules, with consequent extravasation of fluid. Extravasation is believed to be due to
release of substance P. Effective peptide antagonists to substance P have been developed, and
they reduce the extravasation.

Answer. b
33. The receptors responsible for rapid, shallow breathing are

a. Pulmonary stretch receptors

b. J receptors

c. Chemoreceptors

d. Muscle spindles in diaphragm

Solution. B
J or juxtacapillary receptors located at the alveolo capillary junction are stimulated by
- Pulmonary congestion
- Pulmonary edema
- Pulmonary embolism
Stimulation of the J receptors results in
- Apnea followed by rapid, shallow ventilation
- Hypotension
- Bradycardia
Pulmonary stretch receptors are responsible for Hering Breur Reflex.
Chemoreceptor stimulation causes increase in rate and depth of ventilation.
Receptors for tidal respiration are muscle spindles in the diaphragm and intercostal muscles.

Answer. b

34. The double Bohr’s effect is seen in which of the following?

a. Placental circulation

b. Cutaneous circulation

c. Cerebral circulation

d. Coronary circulation

Solution. A
The “double bohr’s effect” helps to increase fetal oxygenation. The transfer of carbon dioxide
from the fetal to the maternal blood shifts the maternal OHDC to the right and fetal curve to the
left, facilitating the transfer of oxygen across the placenta from mother to the fetus.

Answer. a
35. Vitamin essential for transamination is

a. B1

b. B2

c. B6

d. B12

Solution.
C: B6
Vitamin B6 is needed in PLP form as a cofactor for transaminase enzyme.
Various other enzymes where vitamin B6 acts as cofactor are
1. Decarboxylases
2. Deaminases
3. Glycogen synthase
4. Glycogen phosphorylase
5. ALA synthase
6. Kynureninase
7. Cystathione beta synthase
8. Cystathionase

Answer. c

36. A double-stranded RNA genome isolated from a virus in the stool of a child with gastroenteritis was
found to contain 15% uracil. What is the percentage of guanine in this genome?

a. 15

b. 25

c. 35

d. 75

Solution.
c) 35

Answer. c
37. Before being oxidized, fatty acids are activated in the cytosol to form which of the following?

a. ATP

b. CoA

c. Fatty acyl CoA

d. Malonyl CoA

Solution.
(c)Fatty acyl CoA
• Long-chain fatty acidsare activated, in a reaction requiring ATP and CoA, to a fatty acyl
CoA.
• Carnitine reacts with fatty acyl CoA, forming fatty acyl carnitine, in order to transportfatty acid
across the mitochondrial membrane.
Malonyl CoA is an intermediate in fatty acid synthesis.

Answer. c

38. Which among the following is a feature of non competitive inhibition?

a. Increased Vmax

b. Decreased Vmax

c. Increased Km

d. Decreased Km

Solution.
b) Decreased Vmax
• A non competitive inhibitor has no effect on Km. but decrease Vmax
• Vmax : maximum velocity
• Km : the substrate concentration at which the enzyme attains half of the Vmax
Competitive inhibition Non competitive inhibition
Acting on Active site May or may not
Structure of inhibitor Substrate analogue Unrelated
Inhibition is Reversible Generally irreversible
Excess substrate Inhibition relieved No effect
Km Increased No change
Vmax No change Decreased
Significance Drug action Toxicological

Answer. b
39. Pancreatic amylase acts on

a. Terminal α-1,4 glycosidic bond

b. Terminal α-1,6 glycosidic bond

c. Internal α-1,4 glycosidic bond

d. Internal α-1,6 glycosidic bond

Solution.
c: pancreatic amylase acts on internal alpha 1,4 glycosidic linkage.
Glycogen phosphorylase acts on external alpha 1,4 glycosidic linkage of a branch and
pancreatic amylase acts on internal alpha 1,4 glycosidic linkage.

Answer. c

40. α-Linolenic acid is considered to be nutritionally essential in humans BECAUSE:

a. It is an ω3 fatty acid.

b. In humans double bonds cannot be introduced into fatty acids beyond the Δ9 position.

c. In humans double bonds cannot be introduced into fatty acids beyond the Δ12 position.

d. Human tissues are unable to introduce a double bond in the Δ9 position of fatty acid.

Solution. b]
In Human cell , double bond can not be introduced beyond delta 9 position in fatty acid. This
makes linoleic acid and alpha linoleinic acid as essential fatty acid.

Answer. b

41. ACAT is activated by :

a. Apo AI

b. Apo CII

c. free cholesterol

d. Apo B 100

Solution.
c]free cholesterol
ACAT is the intracellular enzyme which is activated by the free cholesterol in the cell.
ACAT converts cholesterol to cholesterol ester which is the storage form of cholesterol.

Answer. c
42. Regarding synthesis of triacyl glycerol in adipose tissue, all of the following are true except

a. Synthesis from Dihydroxyacetone phosphate

b. Enzyme Glycerol kinase plays an important role

c. Enzyme Glycerol 3 phosphate dehydrogenase plays an important role

d. Phosphatidate is hydrolyzed

Solution.
( b) Enzyme Glycerol kinase plays an important role
Adipose tissue does not have glycerol kinase enzyme. So the source of glycerol-3-phosphate for
the synthesis of triacylglycerol is the Dihydroxyacetone phosphate which needs glycerol--
-phosphate dehydrogenase enzyme.

Answer. b

43. A 25-year-old man visits his GP complaining of abdominal cramps and diarrhea after drinking milk.
What is the most likely cause of his problem?

a. Bacterial and yeast overgrowth in the large intestine

b. Infection with the intestinal parasite Giardia lamblia

c. Lack of pancreatic amylase

d. Lack of small intestinal lactase

Solution. d] Lack of small intestinal lactase


Due to lack of lipase in GIT mucosal cell, lactose of the milk is acted upon by intestinal bacteria
which then produces lot of gas in the git lumen leading to bloating, diarrhea.

Answer. d
44. Which one of the following is NOT a phospholipid?

a. Sphingomyelin

b. Plasmalogen

c. Cardiolipin

d. Galactosylceramide

Solution. d] galactosylceramide
Galactosylceramide is a glycolipid and not the phospholipid.
List of phospholipids:
1. Lecithin
2. Cephalin
3. Phosphatidyl serine
4. Phosphatidyl inositol
5. Cardiolipin
6. Plasmalogens
List of glycolipids:
1. galactosylceramide
2. glucosyl ceramide
3. ceramide oligohexoside
4. ganglioside

Answer. d

45. If the ΔG of a reaction is zero:

a. The reaction goes virtually to completion and is essentially irreversible.

b. The reaction is endergonic.

c. The reaction is exergonic.

d. The system is at equilibrium and no net change occurs.

Solution. d]: The system is at equilibrium and no net change occurs.


ΔG°: positive : endergonic reaction
ΔG°: negative: exergonic reaction
ΔG°: zero: equilibrium state

Answer. d
46. Which type of lipase is controlled by glucagon

a. Hormone sensitive lipase

b. Lipoprotein lipase

c. Gastric lipase

d. Pancreatic lipase

Solution.
a] Hormone sensitive lipase
As the name implies, this lipase is regulated by insulin and contra insulin hormones. This enzyme
is active in phosphorylated state. Glucagon activates this enzyme by phosphorylating it in a Camp
dependent manner.

Answer. a

47. Which of the following statement is true regarding “Cyanide” ?

a. Only minimally inhibits the ETC because cytochrome oxidase is the terminal component of the
chain

b. Inhibits mitochondrial respiration but energy production is unaffected

c. Also binds the copper of cytochrome oxidase

d. Bind to Fe+++ of cytochrome a3

Solution. d) Bind to Fe+++ of cytochrome a3


- That is why methemoglobin is an effective antidote since it also has Fe+++.
- Respiration and energy production is the coupled process, so inhibition of one inhibits the other

Answer. d

48. In electrophoresis, if the pH is above isoelectric point, a protein will

a. Precipitate

b. Form zwitterions

c. Migrate to anode

d. Migrate to cathode

Solution.
c: Migrate to anode
At pH above the isoelectric point, the ampholyte will be anion (negatively charged), and anion will
move to anode.

Answer. c
49. A diabetic patient presented with polyuria, his blood glucose was 240mg/dl. Which of these is likely
to occur in this patient of type 1 DM?

a. Increased protein synthesis

b. Increase glycogenesis in muscle

c. Increased conversion of fatty acid to acetyl CoA

d. De-novo cholesterol synthesis

Solution. C: Increased conversion of fatty acid to acetyl CoA

Diabetes is a condition characterized by lack of insulin and in this condition of lack of insulin
there will be
No Protein synthesis
No cholesterol synthesis
No Glycogen synthesis
The increased mobilization, oxidation of fatty acid results in increased formation of acetyl Co A.

Answer. c

50. A patient who suffers from McArdle's disease is most likely to exhibit which one of the following
symptoms

a. Hyperglycemia

b. Enlarged liver

c. Abnormal glycogen structure

d. Muscle cramps

Solution. d) Muscle cramps

Type V (Mc Ardle disease)


- Exercise intolerance
- Muscle cramps
- Myoglobinuria on strenuous exercise
- Increased CK
- Common male preponderance

Answer. d
51. One differentiated cell type is replaced by another cell type is called as?

a. Hyperplasia

b. Hypertrophy

c. Metaplasia

d. Atrophy

Solution. C.
Metaplasia is a reversible change in which one differentiated cell type (epithelial or
mesenchymal) is replaced by another cell type.
It often represents an adaptive response in which one cell type that is sensitive to a
particular stress is replaced by another cell type that is better able to withstand the
adverse environment.

Answer. c

52. A 16-year-old girl undergoes radiologic imaging of her abdomen and is found to have only one
kidney. She had been entirely unaware of this problem. Which of the following terms is most
descriptive of this finding?

a. Agenesis

b. Atrophy

c. Hyperplasia

d. Hypoplasia

Solution. A
The patient has renal agenesis, absence of the kidney due to failure of organ development. The
congenital lack of one kidney differs from atrophy, in which a decrease in the size of an organ
results from a decrease in the mass of preexisting cells.
Unilateral renal agenesis is usually a harmless malformation, and the opposite kidney is often
enlarged due to compensatory hypertrophy.

Answer. a
53. Receptor associated kinase 1 and 3 are involved in which of the following

a. Necrosis
b. Apoptosis
c. Necroptosis

d. Pyroptosis

Solution. C.
Necroptosis:
This form of cell death is a hybrid that shares aspects of both necrosis and apoptosis
- While the entire set of signaling molecules and their interactions is not known,
necroptosis involves two unique kinases called receptor associated kinase 1 and
3 (RIP1 and RIP3).
- Ligation of TNFR1 recruits RIP1 and RIP3 into a multiprotein complex that also contains
caspase-8.
- Caspases are not activated and as in necrosis the terminal events include permeabilization of
lysosomal membranes, generation of ROS, damage to the mitochondria, and reduction of ATP
levels.
- This explains the morphologic similarity of necroptosis with necrosis initiated by other injuries.

Answer. c
54. Ghost cells on microscopy is seen in which of the following?

a. Coagulative necrosis

b. Liqufactive necrosis

c. Apoptosis

d. Autophagy

Solution. A.

Coagulative Liquefactive

Protein degradation Lysosomal acivation and tissue destruction

Architecture maintained Architecture lost

Ghost cells seen No ghost cells seen

Brain infarct, Bacterial infection (suppuration)


Solid organ infarct
Acute pancreatitis

Dry Gangrene Wet Gangrene

Answer. a

55. A routine complete blood count per- formed on a 22-year-old medical student reveals an
abnormality in the differential leukocyte count. She has been complaining of frequent sneezing and
“watery” eyes during the past several weeks and reports that she frequently had such episodes in the
spring and summer. Which of the following cell types is most likely to be increased?

a. Basophils

b. Eosinophils

c. Lymphocytes

d. Monocytes

Solution. B
This type of reaction is primarily mediated by the release of histamine from tissue mast cells, and
the associated cellular infiltrate and peripheral blood findings represent mobilization and
increased numbers of eosinophils. The symptoms reported are those of seasonal rhinitis, better
known as “hay fever,” a manifestation of type I hypersensitivity

Answer. b
56. Function of guardian of genome p53

a. Increase in Cell proliferation

b. Evasion of Apoptosis

c. Inducer of necrosis

d. Reduce cell mutation rate

Solution. D
- TP53, a tumor suppressor gene that regulates cell cycle progression, DNA repair, cellular
senescence, and apoptosis
- Activation of normal p53 by DNA-damaging agents or by hypoxia leads to cell cycle arrest in G1
and induction of DNA repair by transcriptional upregulation of the cyclin-dependent kinase
inhibitor CDKN1A (encoding the cyclin-dependent kinase inhibitor p21) and the GADD45 genes.
- Successful repair of DNA allows cells to proceed with the cell cycle; if DNA repair fails, p53
triggers either apoptosis or senescence.
- In cells with loss or mutations of the p53 gene, DNA damage does not induce cell cycle arrest or
DNA repair, and genetically damaged cells proliferate, giving rise eventually to malignant
neoplasms.

Answer. d

57. Which of the following genes are not involved in hereditary pancreatitis?

a. CFTR gene

b. PRSS1 gene

c. SPINK 1 gene

d. CDH1 gene

Solution. D
Hereditary Pancreatitis
- Hereditary pancreatitis is characterized by recurrent attacks of severe acute pancreatitis often
beginning in childhood and ultimately leading to chronic pancreatitis.
- The disorder is genetically diverse, but the shared feature of most forms is a defect that
increases or sustains the activity of trypsin
- Three genes implicated in hereditary pancreatitis deserve special note: PRSS1, SPINK1, and
CFTR.

Answer. d
58. Which of the following is the most appropriate diagnosis of the kidney shown below?

a. Nephronopthisis

b. Medullary cystic kidney

c. Hydronephrosis

d. ADPKD

Solution. C.
Hydronephrosis is the term used to describe dilation of the renal pelvis and calyces associated
with progressive atrophy of the kidney due to obstruction to the outflow of urine.
Even with complete obstruction, glomerular filtration persists for some time because the filtrate
subsequently diffuses back into the renal interstitium and perirenal spaces, from where it
ultimately returns to the lymphatic and venous systems.
Obstruction
- When the obstruction is sudden and complete, it leads to mild dilation of the pelvis and calyces
and sometimes to atrophy of the renal parenchyma.
- When the obstruction is subtotal or intermittent, progressive dilation ensues, giving rise to
hydronephrosis

Answer. c
59. A 25 year old person presenting with hypo pigmented lesions on trunk, with history of seizures
presents with complaints on abdomen mass and pain. On USG there was a lesion noted in the left
kidney 3X3 cm. What is your most probable diagnosis?

a. Oncocytoma

b. Angiomyolipoma

c. Xp11 translocation associated carcinoma

d. Clear cell RCC

Solution. B.
The person has history suggestive of Tuberous Sclerosis.
Angiomyolipoma
Benign neoplasm consisting of vessels, smooth muscle, and fat originating from perivascular
epithelioid cells.
- Angiomyolipomas are present in 25% to 50% of patients with tuberous sclerosis, a disease
caused by loss-of-function mutations in the TSC1 or TSC2 tumor suppressor genes.
- Tuberous sclerosis is characterized by lesions of the cerebral cortex that produce epilepsy and
mental retardation, a variety of skin abnormalities, and unusual benign tumors at other sites, such
as the heart

Answer. b
60. A 25-year-old man complains of eruptions of blisters on his scalp and inner surface of the groin and
in his mouth. The IF finding of the skin adjacent to bulla is shown below. Which of the following
proteins is targeted by IgG autoantibody in the skin of this patient?

a. Collagen type IV

b. Desmoglein-3

c. E-cadherin

d. Fibronectin

Solution. B.
The diagnosis here is Pemphigus vulgaris. Antibodies aginst demoglein 1 and 3 are the causes.
The clues
1) Blister – skin and mucosa
2) Biospy – suprabasal separation
3) IF – fish net appearance

Answer. b
61. A 25 year old with history of right hypochodrial pain and fever was operated and the specimen is as
shown below. Which of the following is true regarding the below pathology?

a. Fat forty female is a risk factor

b. Seen exclusively in gall bladder

c. Seen in cases of hemolytic anemia

d. Most of the black stones are radiolucent

Solution. C.
The history is suggestive of acute cholecystitis.
Pigment gallstones are brown to black
- Black pigment stones are found in sterile gallbladder bile and brown stones are found in infected
large bile ducts.
- Black stones contain oxidized polymers of the calcium salts of unconjugated bilirubin, small
amounts of calcium carbonate, calcium phosphate
- Brown stones contain similar compounds along with some cholesterol and calcium salts of
palmitate and stearate.
- Approximately 50% to 75% of black stones are radiopaque due to calcium salts

Answer. c
62. What is the defect in the person with the below finding in peripheral smear?

a. Hb Beta chain mutation – 6th postion Glutamine to Valine

b. Defect in enzyme glucose -6- PO4 dehydrogenase

c. Absence of Beta chain of Hb

d. Absence of Alpha chain of Hb

Solution. B
The above picture shows bite cells. It’s a clue for the diagnosis of G6PD deficiency
G6PD deficiency
- G6PD deficiency causes both episodic intravascular and extravascular hemolysis.
- On exposure to oxidant stress the amount of free radicals produced is beyond the capacity of
RBC’s to remove them, resulting in free radical mediated damage to the RBC’s
- Globin chains, which become denatured and form membrane-bound precipitates known as Heinz
bodies.
- The membrane of RBC’s is not stained in the part of the Hb coagulum formed resulting in the
formation of bite cells

Answer. b

63. Incomplete penetrance is seen in which of the following disorders

a. Fragile X syndrome

b. Hemophilia

c. MELAS

d. Marfans syndrome

Solution. D.
Incomplete penetrance is a feature of dominant disorders. Of the four options only Marfans is
autosomal dominant disorder
Patterns of inheritance of the options
a. Fragile X syndrome - XLR
b. Hemophilia - XLR
c. MELAS – mitochondrial
d. Marfans syndrome - AD
Incomplete penetrance and variable expressivity are features of dominant disorders.

Answer. d
64. Which is the most common type of pemphigus

a. Pemphigus vegetans

b. Pemphigus foliaceus

c. Pemphigus vulgaris

d. Pemphigus erythematosus

Solution.
C
Pemphigus vulgaris, by far the most common type
- 80% of all pemphigus
- Oral mucosa; also scalp, face, eye, pharynx, larynx, axilla, groin, trunk; nail involvement  Fatal
if untreated because oral erosions impair swallowing
- Complications are due to staphylococcal infection
Microscopy
- Numerous small, flaccid, suprabasilar bullae with single row of keratinocytes attached to
basement membrane
- Prominent extension of acantholysis into follicular infundibula

Answer. c

65. Which of the following is not the criteria for IgG4 mediated disease?

a. Storiform fibrosis

b. Neutrophil infiltrates

c. Obliterative phelibitis

d. IgG4 levels ? 135mg/dl

Solution. IgG4-related disease (IgG4-RD) is a newly recognized constellation of disorders


characterized by tissue infiltrates dominated by IgG4 antibody-producing plasma cells and
lymphocytes
Criteria on biopsy for IgG4 disease
- Dense Lymphoplasmacytic infiltrate
- Storiform fibrosis
- Obliterative phlebitis
Definite diagnosis of IgG4-RD may be made by demonstrating 1. organ involvement 2. a serum
IgG4 level exceeding 135 mg/dL 3. greater than 10 IgG4+ plasma cells per high-power field and
an IgG4+:IgG+ plasma cell ratio of at least 40% on histologic tissue sections.
The disease spectrum include Mikulicz syndrome (enlargement and fibrosis of salivary and
lacrimal glands), Riedel thyroiditis, idiopathic retroperitoneal fibrosis, autoimmune pancreatitis,
and inflammatory pseudotumors of the orbit, lungs, and kidneys.

Answer. b
66. A 48 year old patient, a known case of Hodgkins Lymphoma was on chemotherapy. Post two cycles
of chemotherapy the size of the lymph node reduced clinically. Follow up biopsy of the patient showed
the below findings. By which of the following mechanisms did the neoplasm primarily responded to
therapy?

a. Phagocytosis

b. Necrosis

c. Inflammation induced damage

d. Apoptosis

Solution. D.
The picture shows multiple cells with dark nucleus and dark eosinophilic cytoplasm.
The question is on chemotherapy response. So we can easily rule out Phagocytosis and
Inflammation induced damage.
Let’s look at the options left – Necrosis and Apoptosis.
- A slide which is necrotic will be completely pink and cell membrane will be lost and you will
have inflammatory cells.
- But the above picture is not having any of the above said features so the answer is Apoptosis.
Apoptosis – Morphology
- Cell shrinks in size and has dense eosinophilic cytoplasm
- Nuclear chromatin condensation followed by fragmentation
- Formation of cytoplasmic membrane blebs
- Breakdown of the cell into fragments (apoptotic bodies – may or may not have nuclear
fragments)
- A lack of an inflammatory response

Answer. d
67. A 25-year-old presented 3 months back with history of acute appendicitis and was operated. Post
op was uneventful and now presented with a history of 1 cm tiny swelling at the incision site. Which of
the following cell will you find which is characteristic of the pathology?

a. Neutrophil

b. Mast cells

c. Giant cells

d. Plasma cell

Solution. C.
To solve this - First let’s come to the clinical diagnosis.
Data provided.
- Past history of surgery and a swelling at the incision site – 3 months later. The history is
suggestive of a suture granuloma. (Foreign Body Granuloma)
- So this is a case of granulomatous inflammation. Now the answer is very obvious, it’s giant cells.

Answer. c
68. A 15-year-old girl presented pruritic over the skin within an hour every time she eats sea food. The
rashes resolve within few hours of the onset. Which of the following immunological mechanism is
involved in this patient.

a. Cell-mediated hypersensitivity

b. Localized anaphylactic reaction

c. Complement mediated reaction

d. Immune complex deposition

Solution. B.
This is a classical history of Type 1 Hypersensitivity. Type I HS reaction
- Food allergies are typically a form of type I hypersensitivity reaction.  The allergens react with
IgE bound to mast cells, mainly in skin and gastrointestinal tract.  More severe allergic reactions
may be systemic and life-threatening from airway obstruction and circulatory collapse
(anaphylactic shock)
- Immediate hypersensitivity may occur as a systemic disorder or as a local reaction.
- The systemic reaction most often follows injection of an antigen into a sensitized individual (e.g.,
by a bee sting), but can also follow antigen ingestion (e.g., peanut allergens).

Answer. b

69. In Alibert bazin syndrome, origin of lymphoma is from

a. Eosinophil

b. B lymphocyte

c. Monocyte

d. T lymphocyte

Solution. d.
Alibert bazin syndrome is the other name of Mycosis Fungoides. Think now the answer will be
much more easy.
Mycosis fungoides (MF) is an epidermotropic, primary cutaneous T cell lymphoma (CTCL)
characterized by infiltrates of small to medium sized T lymphocytes with cerebriform nuclei
Stages
- Patch stage: superficial band-like or lichenoid infiltrate, mainly lymphocytes and histiocytes but
also a few atypical cells (small / medium sized, cerebriform nuclei, halo) usually confined to the
epidermis (basal layer)
- Plaque stage: more pronounced epidermotropism with occasional characteristic intraepidermal
collections of atypical cells (Pautrier microabscesses)
- Tumor stage: more diffuse dermal infiltrate, may lose epidermotropism, more size and shape
variability of tumor cells, histologic transformation may occur ( > 25% large lymphoid cells in the
dermal infiltrate)
IHC
- CD 4, CD3, CD5 AND CD5 POSTIVE
- CD 7, CD 8 - NEGATIVE

Answer. d
70. 2-year-old boy presents with recur- rent infections involving multiple organ systems. Extensive
investigation results in a diagnosis of chronic granulomatous disease of childhood. Which of the
following most closely characterizes the abnormality in this patient’s phagocytic cells?

a. Decreased killing of microorganisms because of enhanced production of hydrogen peroxide

b. Deficiency of NADPH oxidase activity

c. Impaired chemotaxis and migration caused by abnormal microtubule formation

d. Inability to kill streptococci

Solution. B
Chronic granulomatous disease of childhood, a condition characterized by repeated infections and
most commonly X-linked inheritance, is marked by failure of the myeloperoxidase–halide system
of killing within phagocytic cells. It is caused by the deficiency of NADPH oxidase activity. This
results in a secondary deficiency of reactive oxygen metabolites, including H2O2, which, along
with halide ions, functions as a substrate for myeloperoxidase

Answer. b

71. A 65-year-old female is brought to the emergency department after a motor vehicle collision in
which she suffered a leg injury that led to a massive loss of blood. Vitals show a blood pressure of
85/45 mmHg, pulse rate of 112 beats/min, respiratory rate of 21 breaths/min, and temperature of 98.6
F. Hemostasis is achieved, IV lines are maintained, and two liters of normal saline are rushed. She is
scheduled to receive four units of packed red blood cells. An hour after the transfusion is begun, she
complains of flank pain. Repeated vitals show a blood pressure of 105/70 mm Hg, pulse rate of 108
beats/min, respiratory rate of 18 breaths/min, and temperature of 101 F. The transfusion is stopped.
Pink urine is observed in her urine bag. Which of the following is a possible complication of the
patient’s condition?

a. acute hemolytic reaction

b. traumatic injury to kidney

c. Septicemia

d. Anaphylaxis

Solution. A.
- The most likely diagnosis of this patient is acute hemolytic transfusion reaction. Renal failure
and disseminated intravascular coagulation are potential complications.
- An acute transfusion reaction may present very rapidly with fever, oliguria, and hypotension. A
transfusion reaction requires immediate recognition and clinical management. The safest
management is to stop the blood transfusion and
administer saline. All blood transfusion equipment, including the tubing, should be returned to
the laboratory.

Answer. a
72. The cytogenicity of solid tumors is not easily assessed especially in carcinoma cervix because:

a. Metaphase is distinct

b. Due to contamination with infectious agents

c. High mitotic rate

d. Deficient tissue sample

Solution. B
The problems with cytogenetics are:
- Unpredictable growth of the neoplastic cells in tissue culture
- Overgrowth of neoplastic cells by reactive non-neoplastic cells
- Contamrnation of tumor cultures by bacteria or fungi
- Predominance of nonviable tumor (necrotic sample)

Answer. b

73. A 65-year-old female presents for evaluation of fatigue. She states she has recently felt fatigued,
with cold intolerance, and constipation. She is concerned that something is wrong; as years ago, she
used to have constant diarrhea, and now she feels constipated all the time. Physical exam reveals a
small thyroid on palpation. Which of the following is most likely to be present in this patient?

a. Anti-cyclic citrullinated peptide (CCP) antibodies

b. Anti-TSH receptor antibodies

c. Anti-thyroid peroxidase (TPO) antibodies

d. Anti-centromere antibodies

Solution. C
- The final diagnosis of Hashimoto is made by histology.
- The biopsy will show diffuse lymphocytic and plasma cell infiltration.
- Atrophy of thyroid parenchyma is usually present.
- The findings are usually correlated to symptoms and presence of anti-thyroid peroxidase (TPO)
antibodies.

Answer. c
74. A 45 year patient with dementia is having mutation of Presenilin gene, which of the following
mechanisms leads to early onset of Alzhiemer’s disease

a. Accumulation presenilin protein

b. Increased production of APP protein

c. Increased γ-Secretase Activity

d. Increased α-secretase activity

Solution. C.
Presenilin 1 &2 code for γ-Secretase, which ultimately leads to production of Aβ and its
deposition.
APP protein is located on Chromosome 21 and is mechanism for Downs syndrome associated
Alzhiemer’s

Answer. c

75. A kidney biopsy from a patient shows cresentic glomerulonephritis. Which of the following clinical
scenario best fits into this patient?

a. 36 year female with hematuria, proteinuria 2+, rapid declining GFR

b. 5 year old with proteinuria 3+, anasarca

c. 55 year uncontrolled diabetic, proteinuria 2+ and rapid decline in GFR

d. 7 year child with thrombocytopenia, rapidly declining GFR

Solution. A.
Cresentic glomerulonephritis clinically present as Rapidly progressive glomerulonephritis, which
will have features of nephritic syndrome with rapidly declining GFR.

Answer. a

76. All of the following drugs will potentiate the action of d-Tubocurarin Except

a. Neomycin

b. Neostigmine

c. Polymyxin B

d. Clindamycin

Solution. B
d.TC is the skeletal muscle relaxant, neomycin, polymyxin B, clindamycin are having skeletal
muscle relaxing property, whereas neostigmine is the anticholinesterase , improves skeletal
muscle contraction. Neostigmine useful for reversal of action of d.TC.

Answer. b
77. Which one of the following drugs has additional calcium channel blocking property and anti-oxidant
property?

a. Betaxolol

b. Bevantolol

c. Carvedilol

d. Celiprolol

Solution. c
Carvedilol is a b& aBlocker, and also having Anti oxidant, anti-inflammatory and calcium channel blocking
properties, useful for CCF, HT
Beventalol has beta plus alpha blocking action
Betaxolol is the cardioselective beta blocker, useful in glaucoma, safe in asthmatic
patients
Celiprolol- cardioselective beta blocker having NO release, weak β2 agonistic properites

Answer. c

78. All of the following antipsychotic drugs having partial D2 agonistic action except

a. Aripiprazole

b. Cariprazine

c. Brexpiprazole

d. Paliperidone

Solution. D
Paliperidone – active metabolite of risperidone, having 5HT2 antagonistic action ,
approved for schizoaffective disorder
Aripiprazole – HT2 antagonism with partial D2 agonism ( dopamine serotonin stabiliser)
LUMATEPERONE-5HT2block, Partial D1,D2 Agonist
Cariprazine- partial D2,D3agonist
Brexpiprazole- partial D2 agonist

Answer. d
79. which one of the following adverse effect may occur more commonly with oxcarbazepine than with
carbamazepine?

a. Vestibular toxicity ( vertigo)

b. GI toxicity (Nausea, vomiting)

c. Hematological toxicity( aplastic anemia)

d. Hynonatremia

Solution. D
Adverse effects of carbamazepine – Agranulocytosis, Aplastic anemia , Dilusional hyponatremia ,
hypersensitivity reaction(SJS) in patients with (HLA B 1502 gene), Oxcarbazepine,
Esilcarbazepine derivative of carbamazepine causes more hyponatremia

Answer. d

80. All of the following drugs can be used for treating cranial Diabetes insipidus(DI) except

a. Chlorpropamide

b. Carbamazepine

c. Demeclocycline

d. Desmopressin

Solution. C
Demeclocyline is the tetracycline group of antibiotic, causing side effect of diabetes insipidus, so
can be useful for SIADH
For treatiung Central DI- drug of choice is Desmopressin (no role in renal DI)
For treating Lithium induced DI-DOC is Amiloride
Thiazides are useful for treating nephrogenic DI
Drugs causing SIADG are-
SIADH
Chlorpropamide.
MAO inhibitor
Carbamazepine
Phenothiazine
Oxytocin high dose
SSRI
TCA
Vasopressin
Vincristine
Nicotine

Answer. c
81. Which one of the following ARB having thromboxane A2 receptor antagonistic action?

a. Irbasartan

b. Telmisartan

c. Losartan

d. Azilsartan

Solution. C
Losartan- has Uricosuric action, TXA2 antagonism action
Telmisartan- has PPAR gamma agonistic action
Irbesartan can control heart rate useful for heart failure with atrial fibrillation

Answer. c
82. Regarding statin find out the false statement

a. Lovastatin is a prodrug

b. Rosuvastatin is long acting statin

c. Pravastatin not metabolised by CYP enzymes

d. Atorvastatin unsafe in renal dysfunction

Solution. D

PROPERTY STATIN

PRO DRUG Lovastatin , Simvastatin

LONG ACTING Atorvastatin, Rosuvastatin

Pravastatin
LEAST MYOPATHY
Rosuvastatin

CYP3A4 & 3A5 CYP2C9 No hepatic metabolism

Atorvastatin
Lovasatatin Fluvastatin Pravastatin
Simvastatin

PROPERTY STATIN

PREFERRED IN RENAL DYSFUNCTION

PREFERRED IN LIVER DYSFUNCTION

PROPERTY STATIN

11yr & older children

8yr & older children

Answer. d
83. Which of the following drugs, if used chronically, is most likely to increase the toxicity of
acetaminophen?

a. Cimetidine

b. Ethanol

c. Ketoconazole

d. Ritonavir

Solution. B
Ethanol on chronic use indues CYP 2E1 enzyme thereby acetaminophen rapidly converted in a
acetyl benzoquinoimmuno amine (which is the metabolite of acetaminophen), thereby causing
more hepatotoxicity.

Answer. b

84. Which one of the following drugs is the CFTR activator useful in the treatment of irritable bowel
syndrome?

a. Linaclotide

b. Lubiprostone

c. Chrofelemer

d. Asimadoline

Solution. A
Linaclotide. Plecanatide- (CFTR) activator
LUBIPROSTONE- chloride channels activator
Asimadoline is the opioid useful for D-IBS
Crofelemer- is CFTR inhibitor useful for treating HIV drug induced diarrhoea

Answer. a
85. Which one of the following drug useful in gout having higher risk of causing myocardial infarction
and stroke?

a. Allopurinol

b. Febuxostat

c. Lesinurad

d. Rasburicase

Solution. B
Febuxosatat- adverse effects are Hepatotoxic, risk of MI & stroke
Allopurinol adverse effects are- hypersensitivity reactions, TEN, SJS

Answer. b

86. Match the following biologicals with their respective mechanism of action:

a. 1-D 2-C 3-B 4-A 5-E

b. 1-B 2-A 3-E 4-D 5-B

c. 1-E 2-C 3-B 4-D 5-A

d. 1-C 2-E 3-B 4-A 5-D

Solution. A
Explanation-
OMALIZUMAB – Ig E
DUPILUMAB IL-4
MEPOLIZUMAB-IL5
RESLIZUMAB-IL5
BENRALIZUMAB- IL5
TRALOKINUMAB IL-13
LEBRIKIZUMAB -IL13

Answer. a
87. Which one of the following is targeting against NS5b (RNA polymerase) ?

a. Sofosbuvir

b. Boceprevir

c. Daclatasvir

d. Miravirsen

Solution. A
NS5B polymerase inhibitors- Sofosbuvir
NS3/NS4A serine Protease inhibitor-Telaprevir, Boceprevir, Simeprevir
NS5A inhibitors- Daclatasvir, Velpatasvir, Ombitasvir

Answer. a

88. Cefiderocol is approved for

a. Complicated UTI

b. Community acquired pneumonia

c. Complicated intra abdominal infection

d. Skin and skin structure infection

Solution. A
CEFIDEROCOL – complicated urinary tract infection
PLAZOMICIN- useful in Complicated UTIs in adults
ERAVACYCLINE-
complicated intra abdominal infection
OMADACYCLINE-
Community-acquired bacterial pneumonia ,
skin and skin structure infections
SARECYCLINE -
moderate to severe acne vulgaris

Answer. a
89. Dose dependent pancreatitis is the adverse effect of which one of the following anti retroviral
drug?

a. Zidovudine

b. Didanosine

c. Abacavir

d. Stavudine

Solution. B
Didanosine- causes dose dependent pancreatitis
Zidovudine cause myelosuppression
Abacavir- causes hypersensitivity reaction in a patient with HLA B 5701 allele Stavudine- causes
severe Neuropathy, Lactic acidosis, Lipodystrophy

Answer. b

90. Which one of the following anticancer drug useful to prevent post nasal surgical synechiae
formation?

a. Dactinomycin

b. Mitomicin

c. Mitoxantrone

d. Bleomycin

Solution. B
Mitomcin has anti fibroblast action, useful for treating Esophageal stenosis, laryngo treacheal
Stenosis and to To prevent post nasal surgical synechiae formation

Answer. b
91. Mitogen activated protein kinase inhibitor approved for treatment of plexiform neurofibroma is

a. Trematenib

b. Cobimetenib

c. Blimetenib

d. Selumetinib

Solution. D
Mitogen Activated Protein Kinase ( MAPK) inhibitors – useful in malignant melanoma
BRAF V600E inhibitor
Vemurafenib
Dabrafenib
Encorafenib
MEK 1 / 2 INHIBITORS
Trametinib
Cobimetinib
Binimetinib

Selumetinib- Mitogen activated protein kinase inhibitor approved for treatment of plexiform
neurofibroma

Answer. d

92. Which one of the following drug targeting against CD25?

a. Alemtuzumab

b. Daclizumab

c. Rituximab

d. Alefacept

Solution. B
Daclizumab targeting against CD25
Alemtuzumab targeting against CD52
Rituximab targeting against CD20
Alefacept is a recombinant fusion protein of lymphocyte function associated antigen-3 (LFA-3) and
immunoglobulin G dimer that acts to inactive T cells, and is an immunosuppressive agent that was
previously used to treat moderate-to-severe plaque psoriasis

Answer. b
93. Which drug acting on hindbrain and control blood sugar?

a. Dapagliflozin

b. Pioglitazone

c. Acarbose

d. Pramlintide

Solution. D
Pramlintide - Islet Amyloid Poly Peptide(amylin) analogue, acting on hind brain, liver and
intestine , useful for both type 1 &2 DM
Pioglitazone- is the PPAR Gamma agonist, insulin sensitiser
Acarbose- alpha glucosidase inhibitor, useful to control post prandial hyperglycemi
Dapagliflozin is SGLT 2 inhibitor, acting on kidney (PCT), causing glycosuria

Answer. d

94. Which one of the anti obese drug withdrawn because of casuing GI cancer problem?

a. Rimonabant

b. Sibutyramine

c. Lorcaserin

d. Liraglutide

Solution. C
Lorcaserin is 5HT2C agonist, used for obesity, recently withdrawn from market because of
causing risk of gastric cancer
Rimonabant is the inverse agonist of cannabinoid, withdrawn because of causing psychiatry
problem
Sibutyramine is the beta 3 agonist, withdrawn because of causing cardiotoxicity
Liraglutide is the GLP 1 analogue, approved for treatment of obesity

Answer. c
95. Which one of the following drug is useful in the treatment of glaucoma causes hypertrichosis of eye
lash?

a. Timolol

b. Bimatoprost

c. Apraclonidine

d. Dipivefrine

Solution. B
Bimatoprost is the PGF2 alpha agonist causing side effect of hypertrichosis of eyelash
Timolol is the beta blocker causing local side effects of conjunctivitis, systemic problem of
bradycardia, bronchospasm
Apraclonidine a alpha 2 agonist, may cause side effects of drowsiness, lidlag
Dipivefrine prodrug of adrenalin may cause red eye, conjunctival congestion

Answer. b

96. Which of the following is NOT a source of infection in plague?

a. Case of bubonic plague

b. Case of pneumonic plague

c. Infected rodents

d. Infected rat fleas

Solution. A: Case of bubonic plague MC source for bubonic plague is- Infected rat fleas followed
by Infected rodents. Source for pneumonic plague is- Case of pneumonic plague.

Answer. a
97. Diene’s method is used for

a. Mycoplasma

b. Plague

c. Chlamydia

d. Diphtheria

Solution. A: Mycoplasma Colonies can be examined by: a) Hand lens b) Dienes’ staining: Plate is
flooded with alcoholic solution of methylene blue and azure, and examined under low power
microscope. Mycoplasmas retain colour for at least 2 days and appear intense royal blue, whereas
Ureaplasmas appear reddish to greenish blue.

Answer. a

98. A 26 year old male recently underwent renal transplant admitted with bloody diarrhoea. Intestinal
biopsy as shown in the image revealed the pathogen. Choose the correct pathogen.

a. CMV

b. BKV

c. RSV

d. HIV

Solution. A: CMV
• CMV is the most common cause of transplant transmitted infection in post renal transplant
recipients with in first six moths.
• Owl eye inclusion bodies in biopsy is diagnostic feature for CMV

Answer. a
99. A 22 year old female visits her gynecologist complaining of a foul-smelling vaginal discharge and
severe itching. A specimen was collected and examined it by light microscopy revealing highly motile,
nucleated cells with multiple flagella. What is the most likely causative agent of this infection?

a. Balantidium coli

b. Plasmodium falciparum

c. Toxoplasma gondii

d. Trichomonas vaginalis

Solution. D The symptoms are consistent with the sexually transmitted infection caused by
Trichomonas vaginalis. This protozoal flagellate is highly motile and easily distinguished from
other sexually transmitted disease pathogens by light microscopy. The other protozoal pathogens
listed do not cause diseases that present with genitourinary tract symptoms.

Answer. d

100. The definitive host of a parasite is the host:

a. in which asexual reproduction occurs.

b. in which sexual reproduction occurs.

c. that is obligatory for the parasite.

d. that is capable of destroying the parasite

Solution. B Sexual reproduction occurs in the definitive host, whereas asexual reproduction
occurs in the intermediate host. For example, in the case of malarial Plasmodium, the definitive
host is the mosquito, and the intermediate host is the human. In most cases, both hosts are
obligatory for propagation of the parasite.

Answer. b
101. A 45-year-old cattle rancher presents to his physician with a wound on his forearm that resembles
a large scab. Samples collected from the wound were cultured and examined. The bacteria recovered
were Gram positive, nonmotile rods with square ends. The cultured bacteria formed irregularly shaped,
nonhemolytic colonies on blood agar plates and individual cells from the plates had a centrally located
spore. What is the most likely cause of this infection?

a. Listeria monocytogenes

b. Staphylococcus aureus

c. Legionella pneumophila

d. Bacillus anthracis

Solution. D This cattle rancher is suffering from cutaneous anthrax, which is an occupational
hazard. The scab like wound is called an eschar and results from localized edema and
tissue destruction caused by the two toxins produced by Bacillus anthracis. The microbiological
characteristics of the organism are consistent with a diagnosis of B. anthracis infection. The other
microorganisms do not have the chacteristics described.

Answer. d

102. Your patient has episodes of eye tearing, “blood-shot” eyes, and runny nose, which you think may be due
to an allergy to some plant pollen. You refer the patient to an allergist, who performs skin tests with various
allergens. A wheal and- flare reaction is seen on the patient’s back at the site where several pollens were
injected. What is the most likely sequence of events that produced the wheal-and-flare reaction?

a. Allergen binds to IgE on the surface of B cells and IL-4 is released.

b. Allergen binds to IgE on the surface of mast cells and histamine is released.

c. Allergen binds to IgE in the plasma, which activates complement to produce C3b.

d. Allergen binds to IgE in the plasma, and the allergen-IgE complex binds to the surface of macrophages
and IL-1 is released.

Solution. B Type 1 hypersensitivity reaction:

Answer. b
103. Listeria monocytogenes shows which of the following characteristics?

a. It can grow at refrigerator temperatures (4°C).

b. It is an extracellular pathogen.

c. It is catalase negative.

d. It is a gram-negative coccus.

Solution. A
Listeria. monocytogenes grows optimally at 30 to 37°C, but is capable of growth at 4°C. Thus,
refrigeration does not reliably suppress its growth in food. L. monocytogenes is a catalase-
positive, gram-positive, obligate intracellular pathogen. These organisms are found in cattle,
other warm-blooded animals, and fish, where they can cause disease.

Answer. a

104. Which of the following components are found in the cell walls of gram-positive bacteria but not
gram-negative bacteria?

a. Cytoplasmic membrane

b. Lipopolysaccharide

c. Outer membrane

d. Teichoic acid

Solution. D
Gram-positive bacteria have thick, multilayered, peptidoglycan cell walls that are exterior to the
membrane. The peptidoglycan in most gram-positive species is covalently linked to teichoic acid,
which is essentially a polymer of substituted glycerol units linked by phosphodiester bonds. All
gram-positive species also have lipoteichoic acid in their membranes, where it is covalently linked
to glycolipid. Teichoic acids are major cell surface antigens. Gram-negative bacteria have two
membranes—an outer membrane and an inner (cytoplasmic) membrane. Their peptidoglycan
layer is located between the two membranes in the periplasmic space. The periplasmic space also
contains enzymes and various other substances. The outer membrane is distinguished by the
presence of various lipopolysaccharides.

Answer. d
105. Gamma-delta T cells

a. contain very extensive antigen recognition repertoires.

b. express surface markers that are also characteristic of NK cells.

c. generate memory when recognizing antigen on multiple occasions.

d. migrate preferentially to respiratory organs, skin, and peritoneal cavity

Solution. D
Gamma-delta T cells are found predominantly in the respiratory organs, skin, and peritoneal
cavity. Their recognition repertoire is far less extensive that found in alpha-beta T cells. They do
not express significant immunologic memory but do react to antigenic stimuli more rapidly than
do alpha-beta T cells.

Answer. d

106. Lymphoid lineage cells

a. are the most numerous leukocyte population.

b. consist of B, T, and N K cells.

c. contain conspicuous cytoplasmic granules.

d. differentiate from myeloid cell precursors

Solution. B
Lymphocytes including bone marrow-derived (B cells) , thymus-derived (T cells), and natural killer
(NK) cells derive from lymphoid lineage cells. They account for fewer than 40% of blood
leukocytes; neutrophils are the most numerous. Lymphoid lineage cells are agranular leukocytes
and are also poorly phagocytic.

Answer. b
107. A 76-year-old man is diagnosed with Escherichia coli septicemia. The initial immune response to
E. coli (gram-negative bacteria) will include

a. binding by LPS-binding proteins and delivery to receptors on macrophages.

b. formation of specific somatically generated receptors to bind E. coli.

c. generation and secretion of specific antibodies to recognize E. coli.

d. production of E. coli-specific cytokines by lymphocytes.

Solution. A
LPS of gram-negative bacteria is recognized by LPS-binding protein in the bloodstream and tissue
fluids. The LPS-LPS-binding protein complex is then delivered to the cell membrane of a
macrophage, where resident LPS receptors, composed of a complex of proteins (TLR-CD 1 4-M-
-2) bind the bacterial LPS. As a result of receptor engagement, the microbes are ingested and
degraded, the macrophage is activated, and cytokine production and inflammation result. Actions
of somatically generated receptors of B and T cells and of antibodies are part of the adaptive
immune response as opposed to the innate response. Cytokines do not have antigenspecific
activities, and killer activation receptors on NK cells recognize stress-related molecules on the
surfaces of abnormal host cells.

Answer. a

108. In humans, MHC class I I molecules are expressed by

a. all nucleated cells.

b. B cells, dendritic cells, and macrophages.

c. erythrocytes.

d. mast cells.

Solution. B
cells, dendritic cells, monocytes, and macrophages constitutively express MHC class II molecules.
Only a subset of nucleated cells expresses MHC class I I molecules, a n d i t does not include mast
cells o r naive T cells. Erythrocytes do not express M H C class II molecules.

Answer. b
109. An 18-month-old child was brought the pediatrician's office with what appeared to be a sunburn,
although the parents denied that the child had been over exposed to the sun. The parents did recall
seeing an area of redness and small blisters on the child's arm the night before. Which of the following
virulence factors is critical to this disease manifestation?

a. Toxic shock syndrome toxin.

b. Panton-Valentine Leukocidin

c. Protein A

d. Exfoliatin

Solution. D
Exfoliatin is a virulence factor, produced by some Staphylococcus aureus strains, cleaves
desmosomes, resulting in loss of the outer layers of skin. This manifestation is also known as
scalded skin syndrome. The toxic shock syndrome toxin is a superantigen produced by some S.
aureus strains. This toxin causes systemic effects and has been associated with tampon use.
Panton- Valentine Leukocidin is a hemolysin that lyses white blood cells and is produced by many
communityacquired MRSA strains. Protein A is a virulence factor that allows S. aureus to evade
an immune response by binding the Fc region of IgG, resulting in the inverse orientation of the
antibody. Thus, the antibody cannot effectively opsonize the bacterium. The thin microcapsule of
S. aureus is also associated with immune evasion.

Answer. d

110. Which of the following is true of Haemophilus influenzae?

a. Invasive infections are most commonly associated with encapsulated strains.

b. Most invasive infections occur in infants during the neonatal period.

c. Most human infections are acquired from domestic pets.

d. The organism can be readily cultured on sheep blood agar in an environment of elevated CO2.

Solution. A:
Invasive infections are most commonly associated with encapsulated strains The capsule is
antiphagocytic, and facilitates hematogenous dissemination of Haemophilus influenzae. Although
H. influenzae is an important pathogen of infants and young children, passive transfer of maternal
immunoglobulinG may afford neonates protection. Immunity begins to wane in older adults,
increasing the risk of infection for this population. Humans are the only natural host for H.
influenzae. H. influenzae requires both hemin, X factor, and nicotinamide adenine dinucleotide
(NAD), V factor, which are not available in sheep blood agar. Heating the blood lyses the
erythrocytes, releasing both X and V factors, and simultaneously inactivating an NAD inactivating
enzyme present in blood. Media made with such heated blood is termed “chocolate agar.” The
organism does prefer elevated CO2.

Answer. a
111. Which is enrichment media:

a. Selenite F broth

b. Chocolate media

c. Meat extract media

d. Egg media

Solution. a
• Enrichment Medium: Liquid medium (broth) that allows the growth of certain organisms and
inhibits other organisms.
• Selenite F & Tetrathionate broth, Alkaline peptone water.

Answer. a

112. T cell multiplication is stimulated by

a. Bovine serum

b. Phytohaemagglutinin

c. Macrolin

d. Leukotrienes

Solution. B
T cell multiplication is stimulated by phytohaemagglutinin and concanavalin A.

Answer. b
113. Aspergillus is best described by

a. Round black sporangia filled with endospores, sporangia unbranched, rising from a runner
called a stolon from aseptate hyphae

b. Thick walled spores with tubercle like projections all around, on septate hyphae

c. Widespread in environment, conidia may be inhaled microscopic appearance in specimen


reveals dichotomous branching and septate hyphae

d. Rosette like cluster of conidia, on conidiophores, arising out of very thin septate hyphae

Solution. C
Aspergillus species exist only as molds; they are not dimorphic. They have septatehyphae that
form V-shaped (dichotomous) branches.
• These molds are widely distributed in nature. They grow on decaying vegetation,producing
chains of conidia. Transmission is by airborne conidia.A. fumigatus can colonize and later invade
abraded skin, wounds, burns, the cornea,the external ear, or paranasal sinuses. It is the most
common cause of fungal sinusitis.
• In immunocompromised persons, especially those with neutropenia, it caninvade the lungs and
other organs, producing hemoptysis and granulomas.

Answer. c

114. The term “eclipse period” refers to:

a. the period between epidemic outbreaks of diseases that occur in a cyclic pattern.

b. the period between recurrences of disease in individuals with latent virus infections.

c. the time between exposure of an individual to a virus and the first appearance of disease.

d. the time between entry into the cell and disassembly of the parental virus and the appearance
of the first progeny virion.

Solution. D
Following initial attachment of a virus to the host cell, the ability of that virus to infect other cells
disappears. This is the eclipse period. During this period, active synthesis of virus components is
occurring. The time between exposure of an individual to a virus and the first appearance of
disease is referred to as the incubation period (choice C). There is no specific term applied to the
time periods described by A, B.

Answer. d
115. In response to the lipopolysaccharide from a gram negative bacterial infection, local host
phagocytes release proinflammatory cytokines, including I L-6, which then stimulates hepatic synthesis
and release of

a. C-reactive protein.

b. chemokines.

c. complement.

d. immunoglobulins.

Solution. A
IL-6 induces production of C-reactive protein by the liver. It does not induce the liver to produce
chemokines, complement, immunoglobulins, or interleukins.

Answer. a
116. A person has suspended himself by applying ligature around neck so that the point of suspension (knot) is
situated in the occipital region. Such a hanging is called as?

a. Typical

b. Atypical

c. Partial

d. Incomplete

Solution. A.
Typical (Ref: Concise textbook of FMT by RK Sharma 3rd edition Page no 39)
Explanation:
According to position of knot hanging is of two types-Typical hanging and Atypical hanging. In typical
hanging the knot of the ligature should be at the nape of the neck and the knot of the ligature at any site
other than the nape of the neck is Atypical hanging.

Answer. a

117. Viper venom is?

a. Musculo-toxic

b. Neuro-toxic

c. Histotoxic

d. Vasculotoxic

Solution. d) Vasculotoxic (Ref: Concise textbook of FMT by RK Sharma 3rd edition Page no 276)

Explanation: Vipers are hemotoxic, induce blood clotting and hemolysis.

Answer. d
118. Saturnism is a feature of chronic poisoning by?

a. Mercury

b. Lead

c. Arsenic

d. Thallium

Solution. b) Lead poisoning [Ref: Parikh 6th/e p. 9.18]


1. Chronic Lead Poisoning in also known as plumbism or saturnism
2. Lead acetate is also known as salt of Saturn
3. The term “saturnine personality” was introduced by astrologers in the 15th century to describe
people born under the planet Saturn. These people were said to be gloomy (sad) with frequent
headache, fatigue, irritability and depression. For the same reason lead poisoning is also known
as Saturnism.

Answer. b

119. Smoky stool is seen in which poisoning?

a. Mercury

b. Phosphorous

c. Iodine

d. Lead

Solution. b) Phosphorous (Ref: Concise textbook of FMT by RK Sharma 3rd edition Page no 253)
In acute phosphorous poisoning the stool and vomitus may be luminous in dark and may emit
fumes, the phenomenon often called as smoky stool syndrome.

Answer. b
120. Eonism is?

a. Desire to identify with the opposite sex

b. Sexual intercourse with a lower animal

c. Oral sex

d. Obtaining sexual gratification by seeing naked bodies

Solution. a) Desire to be identify with the opposite sex


(Ref: Concise textbook of FMT by RK Sharma 3rd edition Page no 343)
Transvestism or Eonism: A Desire to be identified with the opposite sex. Person finds sexual
pleasure in wearing clothes of opposite sex.

Answer. a

121. Scatologia refers to?

a. Female partner amputates penis of her male partner by sharp object

b. Obtaining sexual stimulation or satisfaction by seeing flames or building destruction.

c. Sexual pleasure is obtained through the compulsive use of obscene language

d. Falling in love with object made by him

Solution. c) Sexual pleasure is obtained through the compulsive use of obscene language
(Ref: Concise textbook of FMT by RK Sharma 3rd edition Page no 350)
Female partner amputates penis of her male partner by sharp object is known as Bobbit syndrome
Obtaining sexual stimulation or satisfaction by seeing flames or building destruction- -Pyromania
Falling in love with object made by him is known as Pygmalionism

Answer. c
122. Passive partner in a sexual relationship where a young boy and man are involved is known as?

a. Catamite

b. Dyke

c. Pederast

d. Sodomist

Solution. A. Catamite
[Ref: The essentials of forensic medicine and toxicology; Dr. KS Narayan Reddy, 33rd edition;
Page no: 427]

Answer. a

123. Rain drop pigmentation is seen in?

a. Lead Poisoning

b. Organophosphorus poisoning

c. Arsenic poisoning

d. Zinc poisoning

Solution. C. Arsenic poisoning


(Ref: Concise textbook of FMT by RK Sharma 3rd edition Page no 258)
CHRONIC ARSENIC POISONING
- Aldrich Mee’s line on finger nails NEET PATTERN
- Rain drop pigmentation on skin (measles like rash)
- Chronic consumption of water containing high arsenic concentrations lead to vasospasm and
peripheral vascular insufficiency –black foot disease NEET PATTERN
- Diabetes polyneuritis (glove and stocking type) gangrene NEET PATTERN
- Canner of skin, lun,g liver (angiosarcoma), bladder and kidney
- Arsenophagists: some people take arsenic daily as tonic or aphrodisiac and acquire tolerance
- Treatment- Chelating agent- Dimercaprol

Answer. c
124. The posture of body as seen in the picture given below is due to?

a. Rigor mortis

b. Cadaveric spasm

c. Pugilistic attitude

d. Post-mortem caloricity

Solution. C. Pugilistic attitude


(Ref: Concise textbook of FMT by RK Sharma 3rd edition Page no 47)
- Heat stiffening occurs when muscle is heated to more than 750C
- Due to coagulation of muscle protein
- Also known as Boxer’s attitude, fencer’s attitude

Answer. c

125. Identify the sign of intrauterine death?

a. Robert’s sign

b. Hyper flexion of spine

c. Spalding’s sign

d. Overcrowding of ribs

Solution. C. Spalding’s sign


(Ref: Concise textbook of FMT by RK Sharma 3rd edition Page no 122)
- The USG image shows over riding and loss of alignment of cranial vault bones – Spalding’s sign.
- Presence of gas shadow in the heart and great vessels—Robert’s sign.
- Earliest sign of maceration is skin slippage.

Answer. c
126. Bruce is related to the following vector relationship with it’s concurrent disease:

a. Aedes mosquito

b. Anopheles mosquito

c. Tse tse fly

d. Mansonia mosquito

Solution. C

Answer. c
127. Lockdown in corona is which type or prevention?

a. Primordial

b. Primary

c. Secondary

d. Tertiary

Solution. B

In case of corona; the COVID19 virus is present in the community so lockdown will prevent the
contact hence prevent the disease

Answer. b

128. Better recall of exposure only among the cases in a case control study can result in:

a. Information bias

b. Confounding

c. Investigator bias

d. Selection bias

Solution. A
TYPES OF BIAS
• Selection/ susceptibility bias: At time of recruitment before data collection
• Sample of study differs from population of interest
• Volunteer based study
• Hospital based study/ Berkesonian bias
• Measurement/ Information/ Misclassification Bias: At time of data collection
• Results in misclassification of disease/exposure
• Recall/ Rumination/ Reporting Bias: Participant Level
• Observer/ Interviewer Bias: Interviewer Level

Answer. a
129. If health policy makers want to evaluate the impact of a prevention program, which is the appropriate
measure to be considered?

a. Period prevalence

b. Incidence

c. Point prevalence

d. Case fatality

Solution. B

Answer. b
130. If a researcher wishes to estimate the incidence of Myocardial infarction cases among a group of
women using oral contraceptive pills followed up for 10year, the researcher has to carry out

a. Case series

b. Cohort study

c. Cross sectional study

d. Ecological study

Solution. B

Answer. b
131. More false positive on screening in a community signify that the

a. Disease has low prevalence

b. Disease has high prevalence

c. Test has high sensitivity

d. Test has low specificity

Solution. A
• More false positives are seen when the prevalence is less (Sn & Sp remain constant)
• This can be proved mathematically by the table given below

Answer. a

132. 75% negative predictivity indicate:

a. 75% population have the disease

b. 25% population tested do not have the disease

c. There is 75% probability that the people tested negative for the disease, do not have the
disease in reality

d. 75% population tested, 75% do not have the disease

Solution. C
• Sensitivity/ Usefullness: Ability of the test to identify correctly those who have the disease/ TP
• Specificity: Ability of the test to identify correctly those who don’t have the disease/ TN
• PPV: If the test results are positive in this patient, what is the probability that this patient has
the disease or in other words, what proportion/ percentage of patients who test positive actually
have the disease in the question
• NPV: If the test results are negative in this patient, what is the probability that this patient
doesn’t have the disease

Answer. c
133. ‘MMR’ the Denominator for Rate is:

a. Numbers of live birth

b. Numbers of females in Reproductive age group in a year

c. Either a & b

d. None

Solution. B
• Maternal Mortality Ratio = Number of maternal deaths within 42 days/ Number of live births * 1
lak
• Maternal Mortality Rate = Number of maternal deaths within 42 days/ Number of females in
reproductive age group * 1 lak

Answer. b

134. Dose of anti – tetanus toxin given to new-born of an unimmunized pregnant female:

a. 250 IU

b. 500 IU

c. 750 IU

d. 1000 IU

Solution. C
• Infants born to unimmunized mothers or partially immunized mothers should be given 750 IU
anti toxin within 6 hours of birth

Answer. c
135. Ideal no. of ANC visits as per WHO is:

a. 8

b. 4

c. 14

d. No Guideline

Solution. C
ANC Visits
• Ideal Visits (WHO):
• Monthly visits till 7th month (7)
• Bimonthly visit in 8th month (2)
• Weekly visit in 9th month (5)
• Minimum Visits: 4 (India); 8 (WHO)
• 1 early registration (< 12 weeks)
• 2nd visit: 14- 26 weeks
• 3rd visit: 28- 34 weeks
• 4th visit: after 36 weeks

Answer. c

136. Indian new-born action plan (INAP) target NMR up to year 2030 is

a. 9/1000 live birth

b. 12/1000 live birth

c. 15/1000 live birth

d. 18/1000 live birth

Solution. A
INAP (India New-born Action Plan)
• Launched in 2014
• Objective: SBR & NMR to single digit by 2030 at national level
• 1st plan to target still birth

Answer. a
137. Which of the following is correctly matched as per categorisation of states under "National
framework for malaria elimination (2016-2030)"?

a. Category 1: States with API of 1 per 1000 and above

b. Category 2: States including all the districts with API of less than 1 per 1000 with some exceptions

c. Category 3: States with API of less than 1 per 1000

d. None of these

Solution. B
National Framework for Malaria Elimination in India (2016- 2030)
Goals
• Eliminate malaria (zero indigenous cases) throughout the entire country by 2030; and
• Maintain malaria– free status in areas where malaria transmission has been interrupted & prevent
re-introduction

Answer. b
138. A researcher measures the height of 100 school going children for his study. What type of
variable is ‘height’?

a. Nominal
b. Ordinal
c. Continuous
d. Discrete
Solution. C
Types of Variables
Quantitative/ Numerical Qualitative

Concept Defines the quantity (kitna) Defines the quality (कैसा)

Measurement Can be measured Directly Can’t be measured directly

Measurement on scale Measured on metric scale Measured on a ordinal / nominal scale

Weight 50, 55, 60, 62, 63 Underweight, Overweight

Height 150, 155, 160, 162, 163 Tall, Short

Blood sugar level 150, 155, 160, 162, 163 Controlled, Uncontrolled

Discrete Continuous

Takes few possible values &


Definition Takes in between value as well
no in between values
Scale for
Nominal/ Ordinal Metric
measurement
Weight, Height, MAC, Temperature (F/C),
No. of children living in the
Examples Blood sugar level, BMI, Hb, Serum
family
cholesterol. Blood pressure (mm Hg)

Binary/ Dichotomous/ Attribute/0-1/ Yes- no data Polytomous

Variable has only two values Variable had > 2 values

Yes/ No, Tall/ Short, Rh +/ Rh - Abo BG, BMI (Under, Normal, Over), BP

Categorical Scales Dimensional

Metric scales (most


Nominal (least preferable) Ordinal
preferable)
Type of
Qualitative Qualitative Quantitative
variable
No specific order, based on Ordered & graded into
Definition Interval & Ratio scale
names categories
Race, Religion, Blood Groups,
Positive or Negative for HIV/ TNM staging, Social
Examples Hepatitis, Names of Hospitals, classes, Severity of BGL, Hb, Serum chl., BP
Type of disease (anaemia), Site disease
of malignancy

Compare Mode, Chi square Median Mean

Reported as Percentage Decimal points


Answer. c

139. Which percentile is equivalent to the median?

a. 25

b. 50

c. 75

d. 100

Solution. B
• Median refers to the 50th percentile
• It is the middle value

Answer. b
140. All the following are true for standard deviation (SD) EXCEPT

a. It is the square root of the average of squared deviations of the observations from arithmetic
mean

b. It is the most important measure of dispersion

c. It is expressed in the same units of measurement as the observation

d. The square of the standard deviation is called mean deviation

Solution. D
Calculation of Standard Deviation:
Step1: Arithmetic mean (X) is written against each individual value in a column (a)
Step2: Deviation of each value from arithmetic mean (X-X) is calculated in column(b)
Step3: Each of the deviation calculated in step 3 are squared and written in column(c)
Step4: Summation (adding) of all squared deviation
Step5: Dividing the number of observations by n and if the sample size is less than30 then
dividing it by n-1
Step6: SD formula

Column (a) Colum (b) Column (c)


Sl No. Haemoglobin (X)
Arithmetic mean (X) Deviation from mean (X – X) (X – X)2

1 13 12 1 1
2 11 12 -1 1
3 11 12 -1 1
4 12 12 0 0
5 12 12 0 0
6 10 12 -2 4
7 10 12 -2 4
8 13 12 1 1
9 13 12 1 1
10 12 12 0 0
11 11 12 -1 1
12 14 12 2 4
13 10 12 -2 4
14 13 12 1 1
15 15 12 3 9
Total (ignoring the sign) 18 32

SD = √32/ 9 = √3.5555555555555 = 1.9

Answer. d
141. What is the RR of acquiring TB in households with a culture-positive case compared with households
without TB?

a. 0.05

b. 0.5

c. 2.0

d. 20

Solution. D

Answer. d
142. Calculate sensitivity:

a. 30/ 60

b. 30/ 50

c. 70/ 100

d. 70/ 90

Solution. A

Answer. a
143. For a PHC the following data for 1 year is given. Calculate TFR

a. 2.3

b. 2.6

c. 2.9

d. 3.2

Solution. C

Answer. c
144. Number of live births in an area is 2000. Maternal mortality ratio has to be calculated for the area
where number of females who died in ANC was 10; during delivery was 30; post partum period 100 died
(10 before 42 days and rest after 42 days till 1 year).

a. 2500/ 1 lakh live birth

b. 7000/ 1 lakh live birth

c. 2500/ 1 lakh population

d. 7000/ 1 lakh population

Solution.

Answer. a

145. You have been asked to design a study for a disease whose prevalence in the community is 10%.
The alpha error has to be kept at 1% with a relative precision of 20% & a power of 20%. What will be
the accurate sample size for this study?

a. 9

b. 20

c. 900

d. 2025

Solution. D
• In this question the alpha error is 1% so confidence level will be 99%
• n = 9pq/ d2
• D = absolute precision (20% of 10% = 2)
• Therefore; n = 9 * 10 * 90/ 2 * 2

Answer. d
146. Type of relationship in the image is:

a. Positive

b. Negative

c. Curvilinear

d. Non linear

Solution. D

Answer. d
147. The vector in the image is found in:

a. Bihar

b. Tamil Nadu

c. Gujarat

d. Not found in India

Solution. D
• The image in the question is that of black fly which causes onchocerciasis and is not found in
India
• Black or dark gray, with gauzy wings, stout antennae and legs, and rather short mouthparts that
are adapted for sucking blood
• Only females bite

Answer. d
148. Which technique is this?

a. Water purification

b. De Fluoridation

c. Salt extraction

d. Sewage segregation

Solution. B
• This is the Nalgonda’s technique of de fluoridation where we add lime first and then alum

Answer. b
149. The contraceptive in the diagram is:

a. DMPA

b. Centrochroman

c. Female condom

d. Emergency Contraceptive pill

Solution. B
• NIRODH: brand of condoms under national program
• Tag line: Achi Aadat Hai (It is a good habit)
• New punch line: Plan Banate Hain
• Injectable contraceptives under national program: Pilot tested in Haryana (Program: Salamati,
all PHC’s, CHC’s, DH will be provided free of cost)
• Expanding the basket of services: injectables (DMPA) - Antara, centrochroman - Chhaya, POP

Answer. b
150. Meta analysis in the image shows a:

a. Protective factor

b. Risk factor

c. Null value

d. Can’t comment

Solution. A
The diamond in the image is showing an OR < 1 so it is protective

Answer. a
151. 28-year-old gentleman presents with bone pains for 2-3 years. His Vitamin D3 levels were
persistently low. He also gives history of frequent micturition and urine analysis had revealed 3+
Glucose. However, his blood sugar levels including Glycosylated Hb was normal on 2 occasions in the
past 6 months. ABG analysis revealed Metabolic Acidosis with High urinary anion gap. The most
probable diagnosis is?

a. Distal RTA

b. Barrter’s syndrome

c. Gitelman’s syndrome

d. Proximal RTA

Solution. d. Proximal RTA


Proximal RTA (Also called Type 2 RTA) presents in young adults with osteoporosis. This is due to
the vitamin d3 deficiency due to its loss in the urine. The common association is Fanconi’s
syndrome which is characterised by a triad of Glycosuria, phosphaturia and Aminoaciduria. All
RTA have Metabolic acidosis with normal plasma anion gap and a high urinary anion gap.

Answer. d

152. 28-year-old gentleman presented with postural hypotension. He has a history of recurrent loin
pain with episodes of hematuria an year back which was investigated and he was diagnosed with
nephrocalcinosis. ABG analysis revealed Metabolic Alkalosis with High urinary Chloride levels. The
most probable diagnosis is?

a. Barrter’s syndrome

b. Gitelman’s syndrome

c. Liddle’s syndrome

d. Gordon’s syndrome

Solution. a. Barrter’s syndrome


It is an AR inhibitory defect of Na+-K+-2Cl- ATPase pump. It typically presents with hypotension
in childhood, but mild mutations can present in adults. Postural hypotension is a sensitive sigh of
dehydration. There is Metabolic alkalosis due to secondary hyper aldosteronism. Defect in
paracellular transport of calcium leads to nephrocalcinosis. Treatment is adequate hydration and
trial of NSAIDs to slow progression of disease

Answer. a
153. The following syndromes are associated with Metabolic Alkalosis EXCEPT?

a. Barrter’s syndrome

b. Gitelman’s syndrome

c. Liddle’s syndrome

d. Gordon’s syndrome

Solution. d. Gordon’s syndrome


Gordons’ syndrome is a AD stimulatory defect of Na+-Cl- cotransporter. It presents with
Metabolic acidosis and hyperkalemia and also referred to as Pseudohypoalderonism II (PHA2).

Answer. d

154. Match the following:

a. A-4, B-3, C-1, D-2

b. A-4, B-2, C-1, D-3

c. A-3, B-4, C-1, D-2

d. A-2, B-4, C-1, D-3

Solution. b. A-4, B-2, C-1, D-3


Renal Osteodystrophy in CKI consists of following with corresponding mechanism involved:
1. Osteitis Cystica Fibrosa : Secondary hyper-parathyroidism
2. Adynamic bone disease : Defect in remodelling.
3. Osteomalacia : Active Vitamin D deficiency
4. Rickets : Acidemia of Type 1 RTA

Answer. b
155. Which one of the following is a Criteria for diagnosis of AKI?

a. Urine output <0.5ml/kg/hr > 12 consecutive hours

b. Urine output <0.3ml/kg/hr > 12 consecutive hours

c. Urine output <0.5ml/kg/hr > 6 consecutive hours

d. Urine output <0.3ml/kg/hr > 6 consecutive hours

Solution. c. AKI can be diagnosed using KDIGO guidelines.


1. urine output <=0.5ml/kg/hr > 6 consecutive hours
2. Rise in S.Creatinine > 0.3mg/dl from baseline < 48hours
3. Rise in S.Creatinine > 1.5*Baseline< 7 days

Answer. c

156. 30-year-old male presented with haemoptysis and RPGN. He is a smoker and does not give history
of precious similar episode. Renal Biopsy reveals Linear Ig deposits on the GBM. Which of the following
statement about the diagnosis is true?

a. Risk of alveolar haemorrhage in this case is unrelated to smoking

b. Plasmapheresis is not indicated in this case

c. Serology is likely to reveal ANCA antibody

d. Prognosis is poor due to high acute mortality

Solution. d. Prognosis is poor due to high acute mortality


Good Pastures syndrome (Pulmonary Renal syndrome) : is an auto-immune disease.
- This is characterized by lung hemorrhage, GN, and circulating antibody to basement membrane,
usually in young men. Hemoptysis may precede nephritis. Lung involvement is more common
among smokers for unknown reasons.
- Rapidly progressive renal failure is typical
- Circulating anti – glomerular basement membrane (GBM) antibody and linear immune –
fluorescence on renal biopsy establishes the diagnosis.
- Serology is likely to reveal Antibody against α-3 subunit of type IV Collagen (Goodpasture’s
antigen)
- Plasma exchange (Plasmapheresis) is indicated to prevent the acute mortality.
- Prognosis remains poor once RPGN occurs.

Answer. d
157. 50-year-old male presented with recurrent painful oral and genital ulcers. He complains of foreign
body sensation in his eyes and has noticed blurring of vison recently. What is the dreaded complication
which can be prevented by early immunosuppression?

a. Stroke

b. Blindness

c. Renal failure

d. Coronary involvement.

Solution. b. Blindness
Bechet’s disease is characterized by a triad of oral and genital ulcers with uveitis. The dreaded
complication is the ocular involvement leading to blindness which can be prevented by prompt
immunosuppression.

Answer. b

158. A 30-year-old male presented with Raynaud’s phenomenon ad hematuria. On examination he has
asymmetrical neuropathy. Laboratory test revealed he is Hepatitis B positive. Renal Angiography
revealed microaneurysm at the bifurcation. What is the diagnosis?

a. Takayasu’s arteritis

b. Fibromuscular dysplasia

c. Polyarteritis Nodosa

d. Microscopic polyangiitis

Solution. c. Polyarteritis Nodosa


Systemic necrotising vasculitis is the other name for Polyarteritis nodosa (PAN) as it causes
fibrinoid necrosis of the medium sized arteries. It is the only arteritis to cause haematuria without
GN due to rupture of microaneurysms (on Renal angiography). Gonadal artery involvement can
mimic acute abdomen. Skin involvement and mono-neuritis multiplex are common associations. It
spares the pulmonary circulation though the bronchial arteries maybe affected in 1-2%of the
cases.

Answer. c
159. Tiffeneau-Pinelli Index < 0.70 is diagnostic of which lung disease

a. Obstructive lung disease

b. Restrictive lung disease

c. Isolated defect of diffusion

d. None of the above

Solution. a. Obstructive lung disease


Tiffeneau-Pinelli index is the ratio of FEV1/FVC on Spirometry.
In Obstructive lung disease, the expiratory volume (FEV1) is predominantly affected leading to
lowering of the ratio. Normal value of the ratio is 0.75. Value lower than 0.70 is specific for
Obstructive lung disease.

Answer. a

160. Which of the following statements regarding Respiratory alkalosis are true EXCEPT:

a. Reduced cerebral blood flow can occur due to reduced PaCO2

b. Salicylates are the most common cause of drug induced respiratory alkalosis

c. It is the most common acid base defect in critically ill patients

d. Increased Progesterone during pregnancy prevents Respiratory alkalosis.

Solution. d. Increased Progesterone during pregnancy prevents Respiratory alkalosis.


Increased Progesterone during pregnancy stimulates the respiratory centre and causes
hyperventilation leading to respiratory alkalosis. Respiratory alkalosis is the most common acid-
base defect found in pregnancy.

Answer. d
161. A 22-year-old man developed painful swelling of his left knee and gives a history of diarrhoea 3
weeks back which resolved in 3 days. Testing for gonorrhoea is negative. Reactive arthritis associated
with the recent enteric infection is suspected. Which of the following is true about this condition?

a. Intestinal infection with amoeba has been associated with this syndrome.

b. There is a higher prevalence of HLA-DQ2.

c. It is more common in women than in men.

d. Synovial fluid shows an elevated leukocyte count

Solution. D. Post enteric reactive arthritis (also known as Reiter syndrome) typically develops
2–4 weeks after an acute diarrheal illness. It is an immune-mediated synovitis, so the synovial
fluid shows an elevated leukocyte count despite the absence of infection. Gonococcal arthritis is
important to exclude in a young person presenting with acute monoarticular arthritis. Shigella sp.
is the most commonly associated enteric organism associated with this syndrome, other causes
include Salmonella sp., Campylobacter jejuni, Yersinia enterocolitica, and even Clostridium
difficile have been implicated.
Amoebic intestinal infections are not associated with reactive arthritis. People who develop this
syndrome have a higher prevalence of HLA-B27 antigen, not HLADQ2, which is associated with
celiac disease.
This syndrome is more common in men and classically is associated with the triad of arthritis,
conjunctivitis, and urethritis.

Answer. d
162. A 33-year-old woman is seen in your office for evaluation of fatigue, malaise, and mild right upper
quadrant discomfort, ongoing for the past 1 month. Her medical history is notable for hypothyroidism,
and her only medication is levothyroxine. Her examination result is notable only for mildly tender
hepatomegaly. On further questioning, she reports no risk factors for viral hepatitis. She does not
consume alcohol or illicit drugs. Her mother has lupus, and a maternal aunt has rheumatoid arthritis.
Laboratory test results are notable for ALT 480 U/L, AST 211 U/L, total bilirubin 0.4 mg/dL, alkaline
phosphatase 89 U/L, total protein 9.1 g/dL, albumin 4.1 g/dL, and TSH 2.1 μIU/mL. Her anti smooth
muscle antibody titre is elevated. Which of the following statements about her likely diagnosis is true?

a. This condition most often occurs in men.

b. Treatment of choice is prednisone with azathioprine.

c. There is a very low risk of developing cirrhosis.

d. It does not recur after liver transplant

Solution. B. Treatment of choice is prednisone with azathioprine.


Patient’s elevated hepatic enzymes, female sex, elevated globulin levels (total protein, albumin),
lack of risk factors for chronic viral hepatitis, personal history of hypothyroidism that could be
autoimmune in nature, family history of autoimmune disorders, and high-titre anti–smooth muscle
antibody test are strong clues suggesting autoimmune hepatitis.
Autoimmune hepatitis is more common in women. Its Treatment consists of a glucocorticoid (such
as prednisone) with azathioprine.
Undiagnosed and untreated autoimmune hepatitis may lead to the development of cirrhosis.
Autoimmune hepatitis recurs in up to 50% of patients after liver transplant despite the
immunosuppression regimen.

Answer. b

163. A 62-year-old man with chronic hepatitis C–associated cirrhosis presents to the emergency room
with confusion and abdominal pain. He is noted to have ascites and asterixis. Which statement is true
regarding care of this patients?

a. Patients with ascites fluid total protein <1.1 g/dL and serum bilirubin >2.5 mg/dL should
receive prophylactic antibiotics as risk of SBP is high.

b. Endoscopic variceal sclerotherapy is the preferred option for secondary prophylaxis of variceal
haemorrhage.

c. If Model for End-Stage Liver Disease (MELD) score more than 10 consider liver transplant.

d. All are true

Solution. A. Patients with cirrhosis who have low total protein levels in ascites fluid and
advanced liver disease are at increased risk of developing spontaneous bacterial peritonitis and
should therefore receive prophylactic antibiotics.
A combination of a nonselective beta blocker plus endoscopic variceal band ligation (not
sclerotherapy) is the preferred method for secondary prophylaxis of variceal hemorrhage.
Any MELD score can be listed for Liver transplant.

Answer. a
164. A 26-year-old woman with prior history of IV drug abuse resulting in hepatitis C coinfection is
seen in a prenatal clinic for routine care. She is in the third trimester of pregnancy with her first child.
Which of the following statements regarding transmission of hepatitis C virus (HCV) to neonate is true?

a. Caesarean section reduces the risk of transmission to a greater degree than vaginal delivery.

b. Risk of transmission is 5% to neonate.

c. Breastfeeding is associated with very high risk of transmission.

d. Patients with HCV genotype 1 have a greater risk of transmission than other genotypes.

Solution. B. The vertical risk of transmission is 5%.


Breastfeeding is not associated with high risk of transmission. The mode of delivery (vaginal
versus caesarean section), and viral genotype are not associated with increased risk of vertical
transmission of hepatitis C virus.

Answer. b

165. A 32-year-old female with past history of rheumatic fever (20 years back) that was treated with
antibiotics. She is asymptomatic. Her physical examination shows a heart rate of 70 beats per minute
with a blood pressure of 122/77 mm Hg. She has no jugular venous distention. Her lungs are clear. She
has a regular rate and rhythm with a loud S1. A soft early diastolic rumble is heard at the apex with an
opening snap after S2. There is wide separation between A2 and the opening snap. No peripheral
edema is present. Which of the following is the most appropriate next recommendation for this patient?

a. Observation.

b. Start benzathine penicillin.

c. Start diuretics.

d. Refer for balloon valvotomy.

Solution. B. The patient’s examination is consistent with rheumatic mitral stenosis. Severity of
her mitral stenosis is mild based on her examination, and she is asymptomatic. Current guidelines
recommend antibiotic prophylaxis for secondary prevention in patients with rheumatic fever and
carditis. Diuretic therapy is recommended for patients with symptoms of heart failure.

Answer. b
166. A 48-year-old man presents with dyspnoea and dizziness. On examination, there is paradoxical S2
split. A fourth heart sound is present at apex, there is ejection systolic murmur at the base which
increases in intensity with the Valsalva manoeuvre and with standing. What is the most likely
diagnosis?

a. Dilated cardiomyopathy

b. Mitral valve Prolapse

c. Aortic stenosis

d. Hypertrophic obstructive cardiomyopathy

Solution. D.
- Ejection systolic murmur at base can be due to HOCM or Aortic stenosis. Here murmur becomes
louder with manoeuvres that decrease preload (Valsalva manoeuvre and standing), a cardinal
physical examination finding present in hypertrophic obstructive cardiomyopathy.
- Dilated cardiomyopathy can present with a third heart sound and murmurs of mitral
regurgitation.
- MVP shows non ejection click and late systolic murmur at apex that increases with Valsalva and
standing.
- AS murmur will be heard at aortic area and will decrease with Valsalva and standing.

Answer. d
167. A patient presented with palpitations associated with dyspnea. The ECG showed these findings.
What is the most likely cause?

a. Sinus tachycardia

b. PSVT

c. Atrial fibrillation

d. Atrial flutter

Solution. C. Narrow QRS complex +absence of P waves + irregular heart rate favours diagnosis
of atrial fibrillation.

Answer. c
168. Theme and Focus: Chest pain
Option list: Each option list can be selected once, more than once or none.
a. Stable angina
b. Unstable angina
c. Myocardial Infarction
d. Aortic dissection
e. Acute pericarditis
f. Pleuritis
Case 1. A 69-year old man complains of retrosternal pain that radiates to his left hand and sweating for one
hour.
Case 2. A 67-year old man complains of sudden, severe and tearing chest pain that radiates to interscapular
region.
case 3. A 37-year old man complains of retrosternal severe chest pain in rest that remission occurred
spontaneously after 20 minutes.
case 4. A 35-year old man complains of sharp chest pain that relieves with sitting and leaning forward

a. case 1-c,case 2-e,case 3-a,case 4-f

b. case 1-c,case 2-a,case 3-b,case 4-e

c. case 1-c,case 2-d,case 3-b,case 4-e

d. case 1-c,case 2-d,case 3-a,case 4-e

Solution. C. The different characteristic of chest pain are

Answer. c
169. About 3 weeks after diarrhoea, 18-year-old boy complains of weakness of his both lower limbs.
Over several days, weakness progress to include his upper limbs also. On physical examination he has
weakness in all 4 limbs, absent reflexes; CSF examination showed increase in protein only most likely
diagnosis is

a. Amyotropic lateral sclerosis

b. Guillian Barrie Syndrome

c. Bell's palsy

d. Multiple sclerosis

Solution. B
Post-diarrhoea paraplegia can be due to transverse myelitis or GBS.
Absent reflexes with albumin-cytologic dissociation in the CSF is characteristic of GBS.

Answer. b

170. A 58-year-old woman is seen for complaints of very sharp pain lasting about 1 minute over her
right cheek and lips. These pain episodes occur in clusters with intense pain during the episode. When
an episode occurs, it is present both day and night and can recur over a period of about a week.
Paroxysms of pain can be elicited by washing her face. On physical examination, there is no sensory or
motor loss in the right face. There are no masses. Touching the right face does bring about an episode
of pain for the patient. What is the next best step in management of this patient?

a. Initiate treatment with Carbamazepine

b. Temporal artery biopsy

c. Microvascular decompression surgery

d. Nerve conduction study

Solution. A
The features are characteristic of Trigeminal Neuralgia. The initial treatment is palliative with
Carbamazepine. If MRI shows vascular loop, microvascular decompression surgery maybe done to
offer potential cure.

Answer. a
171. A 32-year-old woman presents for neurologic evaluation after experiencing a severe burn on the
palm of her right hand. She had placed her hand onto the hot surface of a smooth electric range. She
did not feel the burn when it occurred, and only when she picked her hand up did she notice the burn.
After that, it was discovered that the patient unknowingly has bilateral loss of pain and temperature
sensation in both hands. However, she does have touch and vibratory sense. Mapping of her loss of
sensation shows following pattern. What is the most likely diagnosis?

a. Neoplastic spinal cord compression

b. Subacute combined degeneration

c. Syringomyelia

d. Transverse myelitis

Solution. C
Cape and Hood anaesthesia is typical of syringomyelia.

Answer. c

172. Which of the following focal motor seizure is often refractory to medical therapy?

a. Jacksonian march

b. Todd’s paralysis

c. Epilepsia partialis continua

d. All of the above

Solution. C
NO EXPLANATION GIVEN

Answer. c
173. Which of the following about focal seizures with impaired awareness is false?

a. Following seizure patient may develop aphasia

b. Motionless stare denotes beginning of ictal phase

c. Retrograde amnesia following the seizure

d. Impaired awareness is usually accompanied by Automatism

Solution. C
NO EXPLANATION GIVEN

Answer. c

174. Identify the true statement regarding the condition shown in the following Xray:

a. Majority cases are due to trauma

b. It is a surgical emergency

c. Intercostal drainage tube placement is indicated

d. Pleurodesis must be performed to prevent recurrence in all cases

Solution. c. Intercostal drainage tube placement is indicated


The X-ray shows Hydropneumothorax. Majority cases are iatrogenic secondary to pleural tapping.
It needs elective ICD tube placement and pleurodesis is not routinely recommended.

Answer. c
175. Identify the phenomenon in DM and choose the correct statement/s about it.

1. The picture shows Dawn phenomenon


2. It is due to excess night-time dose of Insulin
3. Patient needs Increase in night-time dose of Insulin
4. The phenomenon is physiological due to GH surge in early morning hours

a. 1 & 2 are true statements

b. 2 is the true statement

c. 3 & 4 are true statements

d. 4 is the true statement

Solution. B. 2 is the true statement.


The image shows Somogyi phenomenon which is due to excess night-time dose of insulin.

Answer. b
176. A young boy presents with the lesion as shown. He has been having this lesion since birth. A
doctor tried to burn it with cautery but the lesion recurred. Pick the true statement:

a. Biopsy needs to be done to check for malignancy

b. Aggregation of melanocytes is expected

c. Aggressive management is required

d. Sun exposure has a role

Solution. B
The diagnosis is congenital melanocytic nevus (CMN). CMN which is small (as in the pic) does not
have a premalignant potential. Large CMN (> 20 cm) will have a premalignant potential. On
biopsy, CMN has aggregations of melanocytes. Management for CMN is only for cosmetic reasons
and overaggressive removal causes more scarring and disfigurement. Sun has no role.

Answer. b
177. A patient presents with asymptomatic hypo pigmented lesions on chest and back. His woods lamp
is as shown. Pick the true statement:

a. Autoimmune disease

b. Smear would help in establishing the diagnosis

c. Griseofulvin would help in resolution

d. Biopsy would be needed to establish the diagnosis

Solution. B
Yellow fluorescence is shown on the back in this pic. This would be seen in Pityriasis versicolor.
Smear shows ‘spaghetti and meatball’ appearance. It is not an autoimmune disease. P. versicolor
is treated with azoles and not Griseofulvin which is ineffective. Biopsy is not required and a
simple smear is enough for diagnosis.

Answer. b
178. A young male comes with lesions all over his body since 15 days. On examination, erythematous
papules and scaly lesions are noticed on his trunk and limbs as shown. He is not experiencing any
symptom in the lesions. He also reports perianal lesions. What is the next investigation to be done?

a. Skin biopsy

b. Scrapping of scales for KOH mount

c. VDRL

d. Urine for microscopy

Solution. C
Asymptomatic erythematous papular and scaly lesions on trunk and also classically on acral areas
(soles and palms) in a young male strongly suggests secondary syphilis. Perinala lesions would be
condyloma lata. VDRL is positive in 100% of patients with secondary syphilis.

Answer. c
179. A patient of recurrent Type 2 Lepra reaction is on treatment. She now reports darkening of her
skin as shown. The likely culprit drug is:

a. Dapsone

b. Rifampicin

c. Clofazimine

d. Thalidomide

Solution. C
Clofazimine treatment causes appearance of reddish-blue or brown pigmentation and is
concentrated in the lesions of patients with Hansen’s disease. This pigmentation is a cause of
noncompliance with this drug in the treatment of Hansen’s disease. Histologically, a periodic
acid–Schiff-positive, brown, granular pigment is variably seen within foamy macrophages in the
dermis. This has been called “drug-induced lipofuscinosis.”

Answer. c

180. An 8 year old child presents with localized alopecia over scalp with scales. The diagnosis is:

a. Alopecia areata

b. Trichotillomania

c. Tinea capitis

d. Lichen planus

Solution. C
Child age group would support T. capitis diagnosis. Also, localised scaly plaques (grey patch)
would be in T. capitis. Alopecia areata would not be scaly. Trichotillomania is unusual in children
also would not have scaling. Lichen planus is also not scaly but has scarring localised patches.

Answer. c
181. A 60-year-old patient comes with the following nail changes. Pick the true statement:

a. Microsporum will be the commonest cause

b. Extensor elbow plaques are expected

c. Oral Terbinafine is the drug of choice

d. Associated with alopecia

Solution. C
The image shows yellow discoloration of the lateral and distal edge of the nail. This would be in
onychomycosis. Trichophyton rubrum is the m/c cause. Oral Terbinafine is the DOC

Answer. c

182. Koebner’s phenomenon is not seen in

a. Molluscum

b. Darier disease

c. Lichen planus

d. Contact leucoderma

Solution. D

Answer. d
183. Lesions occurs on the palm and lip since 5 days as shown. The diagnosis for the left sided image
would be:

a. Fixed drug eruption

b. Erythema multiforme

c. Pemphigus vulgaris

d. Stevens Johnson syndrome

Solution. B
The left sides image is of a target lesion which is commonly associated with HSV-1 (causing
herpes labialis- as shown in the right side image). Fixed drug eruption does not have target
lesions and grouped vesicles as shown. PV would have raw lesions, flaccid vesicles, and mucosal
involvement. SJS would have Targetoid lesions and mucosa involved.

Answer. b
184. A 65-year-old male who was operated, was put on Hyoscine for his post-operative nausea and
vomiting. The next evening, the attenders started complaining of excessive sleep in the morning and
decreased sleep in the night, gesticulating to self, talking to self and restlessness with evening worsening
of symptoms. On examination, the patient was oriented to place and person but not time. The drug of
choice for the above condition is

a. Lorazepam

b. Haloperidol

c. Fluoxetine

d. Aripiprazole

Solution. b
Solution: History is suggestive of Delirium.
Delirium:
● Also known as ICU-psychosis, organic psychosis, acute brain failure
● It is acute confusional state
● Always a precipitating factor will be present
● Precipitating factors:

● Clinical features:
› Disturbances of consciousness
› Poor attention and concentration
› Disorientation to time, place and person
› Alteration of sleep-wake cycle
› Evening worsening of symptoms (Sundowning phenomenon)
› Floccillations/carphologia: picking behaviour of bed sheets
› Emotional lability
› Restlessness
● Neurotransmitter involved: ↓ Acetylcholine
● Structure involved: Reticular Formation
● Treatment:
› Treatment of underlying cause
› DOC: Haloperidol
Reference: Kaplan and Sadock’s Synopsis of Psychiatry, 11th edition, Page 697-703

Answer. b
185. A known alcoholic was brought to the Emergency Room with complains of palpitations and
suspiciousness. On examination, tremors are seen and PR of 122/min but the patient was oriented to
time, place and person. Next step of management is:

a. Benzodiazepines

b. Anticraving agents

c. Aversive agents

d. No intervention needed

Solution. a
Solution: The history is suggestive of alcohol withdrawal state without convulsions or delirium
tremens
● In case of Alcohol Withdrawal State, the DOC: Benzodiazepines
● Preferred are long acting BZDs like chlordiazepoxide as short acting BZDs like lorazepam can
precipitate severe withdrawal if a dose is skipped.
Reference: Kaplan & Sadock’s Synopsis of Psychiatry, 11th edition; Page 636-637

Answer. a

186. A 20-year-old male presented with head and neck twisted to one side to ER. History revealed that
the patient was given Inj Fluphenazine the previous night. Drug of choice is:

a. Inj Haloperidol

b. Inj Lorazepam

c. Inj Promethazine

d. Tab Escitalopram

Solution. c
Solution: Acute Dystonia:
● It is the 1st EPS to develop- occurs within hours of administering antipsychotics
● Clinical features: Muscle spasm in any part of the body
› Oculogyric crisis: Eye rolling upwards
› Torticollis: Head and neck twisted to one side
› Laryngeal spasm→ death
● Prevalence: approximately 10% but more common
› in young males
› with high potency drugs (e.g., haloperidol)
● Treatment: Doc: Anticholinergics
› i/v promethazine may be given
› i/v diphenhydramine can also be given
Reference: The Maudsley Prescribing Guidelines in Psychiatry, 13th edition; Page 91-92

Answer. c
187. A 30-year-old male was diagnosed with Major Depressive Disorder and was prescribed on Tablet
Escitalopram 10 mg Once Daily dosage. Most common long term side effect is:

a. Anxiety

b. Nausea

c. Delayed ejaculation

d. Rebound depression

Solution. c
Solution:
Adverse effects of SSRIs:
› m/c: Upper GI side effects (through 5HT3 receptor)- nausea > diarrhea
› QTc prolongation
› Headaches
› Paradoxical anxiety (5HT2C action)
› Insomnia and somnolence
› Restless leg syndrome
› m/c long term side effect: Sexual Dysfunction [anorgasmia, inhibited orgasm, decreased libido]
Reference: Kaplan & Sadock’s Pocket Handbook of Psychiatric Drug Treatment, 7th edition; Page
199-202

Answer. c

188. A middle aged female patient was diagnosed to have OCD. The defence mechanism not seen in
above condition is:

a. Undoing

b. Reaction formation

c. Projection

d. Isolation

Solution. c
Solution:
Defence mechanisms in OCD:
● Isolation
● Reaction formation
● Undoing
Reference: New Oxford Textbook of Psychiatry, 2nd edition; Page 768

Answer. c
189. A 35-year-old male who is a known case of psychosis on regular treatment, developed disturbed
vision. On examination, it revealed retinal pigments. The most probable causative agent is:

a. Chlorpromazine

b. Ziprasidone

c. Thioridazine

d. Olanzapine

Solution. c
Solution:
Ocular side effects of antipsychotics:
● Chlorpromazine→ deposits in anterior portions of the eye (cornea and lens)
● Thioridazine→ only antipsychotic to cause retinal deposits, which in advanced cases can
resemble retinitis pigmentosa → ⸫ maximum daily dose of thioridazine has been limited to 800
mg/day
Reference: Katzung’s Basic and Clinical Pharmacology, 12th edition; Page 512

Answer. c
190. Interpret the following findings on sleep study:
EEG- low-voltage, mixed frequency activity; saw tooth waves, theta activity
EOG- activity +
EMG- Tonic atonia with phasic twitches

a. Awake adult

b. Awake child

c. REM sleep

d. NREM sleep

Solution. c

Answer. c
191. A young female presented with repeated actions of bathing. On probing further, she revealed that she was getting
repeated thoughts of contamination and had to bath or otherwise would lead to anxiety. The above disorder is linked to
which stage of Psychosexual development?

a. Oral
b. Anal
c. Phallic
d. Latency
Solution. b
Solution: The above history is suggestive of OCD
Sigmund Freud’s Psychosexual Stages of Development:
Answer. b
192. suture of choice for abdominal closure :

a. Polygalactin
b. Polydiaxone
c. Plyamide
d. Catgut
Solution. b
Answer. b

193. most important prognostic factors for burns:

a. asphyxia

b. age

c. bsa

d. degree of burn

Solution. B:
AGE >> BSA ARE PROGNOSTIC FACTOR ..( REVISED BAUX SCORE)

Answer. b

194. integra is biological dressing dereived from:

a. shark chondroitin

b. human acellualr dermis

c. collagen

d. porcine gut

Solution. B
biological dressings :
a. Integra : from shark chondroitin
b. Alloderm: acellualr human dermis
c. Biobrane : porcine derivative
d. Transcyte : collagen fibroblast derivative

Answer. b
195. revised fluid requirement for pediatric burn according to atls:

a. 2ml/kg/bsa

b. 4 ml/kg/bsa

c. 3 ml/kg/bsa

d. 6 ml/kg/bsa

Solution. c
fluid requirement per day :
adults : 2ml/kg/bsa
children : 3ml/kg/bsa
pediatric :4ml/kg/bsa

Answer. c

196. identify the instrument shown in the image :

a. lumbar puncture needle

b. trocar

c. veress needle

d. laparoscopic aspiration needle

Solution. c.
#this is Veress needle used for creating pneumoperitoneum
# V Neddle is introduced like a dart at 30 degree to one wall and perpendicular to
one wall
#maximum flow rate via veress is 2-3l/min

Answer. c
197. preferred management of intractable peptic ulcer disease :

a. H .pylori therapy (long duration)

b. highly selective vagotomy

c. Truncal Vagotomy and GJ

d. Antrectomy and Vagotomy

Solution.
b.
#highly selective vagotomy is nowdays the preferred management of intractable
PUD.
# Other surgeries :
@trucal vagotomy and GJ
@antrectomy and vagotomy

Answer. b

198. all except are false for mentrier’s disease:

a. Antrum is always involved

b. giant mucosal folds

c. hyperproteinemia

d. hyperchlorhydria

Solution. b
Menetriers disease is protein loosing hypertrophy .
#there are giant gastric folds with atrophy of parietal cells, hence achlorhydria
#antrum is spared
#premalignant in nature
#1st line management is Protein replacement and Cetuximab
#gastrectomy

Answer. b
199.
identify the knot being performed

a. granny’s knot

b. reef’s knot

c. Meltzer’s knot

d. Aberdeen’s knot

Solution. d
# loop out of loop interlocking followed by termination . This is Aberdeen termination .
#Meltzers knot is slip knot used for extra corporeal suturing for laparoscopic knotting
#reef is 2 throw square knotting
#granny is 2 throw square knotting

Answer. d

200. all except are true for achalasia:

a. pressurization of esophagus

b. high amplitude wave forms

c. failure of LES to relax

d. simulatneous mirrored contractions

Solution. b
# pressurization of esophagus
# low amplitude wave forms
# failure of LES to relax
#simulatneous mirrored contractions

Answer. b
201. all except are true for short bowel syndrome :

a. associated with small bowel length <200 cm.

b. teruglutide is useful medical management

c. atresia is the mc cause in children

d. Bianchi’s procedure is the 1st line management

Solution. d
1st line management is conservative management with PPI/ Bile acid sequestering agents
/mparenteral nutrition
#lengthening procedures are indicated if the medical management fails :
Bianchi’s sx= LILT =longitudinal incision and lengthening procedure .
STEP: serial transverse enteroplasty procedure

Answer. d

202. All except are indications for parenteral nutrition :

a. Crohns

b. Pancreatitis

c. Burns

d. entero-cutaneous fistula

Solution. B
# Pancreatitis : enteral nutrition is preferred
indication for parenteral nutrition :
a. Severe malabsorption
b. Entero-cutaneous fistula
c. Short bowel syndrome
d. Burns
e. Crohns( in high type intestinal bypass)
f. Contradications for enteral nutrition

Answer. b
203. intra abdominal hypertension is defined as IAP:

a. >8 mm Hg

b. >10 mm Hg

c. >12 mm Hg

d. >20 mm Hg

Solution. IAP :
Normal >5-7mm Hg
IAP >12mmHg
ACS :
IAP> 20mmHg
+
New Onset Organ Failure
+
abdominal Perfussion pressure <60mmHg

Answer. c
204.
blend mode 2 of monopolar electrosurgery refers to:

a. 50% on and 50 %off cycle

b. 40% on and 60 %off cycle

c. 25% on and 75 %off cycle

d. 6% on and 94 %off cycle

Solution. B

Answer. b
205. 26 year male brought to ER in unconscious state with pulse :120bpm ; BP: 90/60 ; with flat neck
veins . The patient had labored breathing with decreased breath sounds on the left side. Heart was
normal at auscultation. The definite treatment for the following scenario :

a. Needle decompression @2nd ICS

b. USG guided pericardiocentesis

c. Needle decompression @5th ICS

d. tube thoracostomy @5th ICS

Solution. d
tube thoracostomy @ 5th ICS is definite management for the condition described.
This is case of tension pneumothorax with the unstable status of the patient.The 1st line
management for unstable patients is :needle decompression .
And the definite management is Tube insertion at 5th ics

Answer. d

206. all except are true statements regarding appendicitis :

a. more common in young males

b. pelvic type is most common variant

c. CECT is definite incestigation in case of doubt

d. laparoscopic appendicectomy is gold standard in children

Solution. b
#retrocecal is the mc type .
# CECTis the definite investigation
#acute appendicitis is more prevelant in young , males
#Laparoscopic appendicectomy is gold standard for children nowdays

Answer. b
207.
all except are use for the following blade :

a. Incision and drainage

b. Venotomy

c. Suture removal

d. Port creation in laparoscopy

Solution. c
# the blade shown in image is no.11 which is not used for suture removal

Answer. c

208. all except are true for carcinoids :

a. Midgut is the mc site of incidence

b. soft , friable and bleed to touch

c. size is most important prognostic factor

d. urine 5 HIAA is diagnostic

Solution. b
# Carcinoids are hard , nodular ,yellow color structures
#size is the mi prognostic factor
#they are APUD’omas which secrete vaso-actice peptides that breakdown and
released in urine as 5HIAA
#Mid gut is mc site. ( order : Appendix>>ileum>>rectum-colon>bronchus)

Answer. b
209. maximun permissible length : width for random position flap:

a. 2:1

b. 3:1

c. 4:1

d. 5:1

Solution. B
#maximum permissible L:w ratio in Random position flaps is 3:1
examples of RPF ☺
a. Transposition Flap : subtypes : z plasty ; rhomboid flap ( limberg flap)
b. Rotation Flap
c. Advancement flap : subtypes : VY flap ; Rectangular flap
d. Interposition Flap : eg: thenar flap

Answer. b
210. Stemmer sign is seen in :

a. varicose ulcers

b. peripheral vascular disease

c. beurgers disease

d. filariasis

Solution. D :
filariasis..
#stemmer’s sign is inability to pinch the skin ( Seen in grade 2 filariasis)

Answer. d
211. preferred diagnostic modality for suspected diaphragmatic injury :

a. FAST

b. CECT

c. diagnostic laparoscopy

d. HRCT thorax

Solution. c
# IOC for suspected diaphragm injury is laparoscopy as it is both diagnostic and therapeutic
# diaphragmatic injury is more common with penetrating injury at or below 5th ICS.
#mostly asymptomatic
#all diaphragmatic injuries require definite repair

Answer. c

212. 26 year old male presenting to ER with head injury . There is RIGHT side pupil dilation with eyes
opening to pain .patient is moaning in pain and shows withdrawal to pain. Calculate GCS-P for the
patient :

a. 8

b. 7

c. 9

d. 10

Solution. B
# GCC – pupil score = GCS pupil score:
calc:
eye opening to pain : e2
moaning: V2
withdrawal to pain : m4
pupil score :
both dilated : 2
single dilated : 1
none dilatd : 0
gcs-p : 8-1=7

Answer. b
213.
26 yaer old female presenting with renal colic ; recurrent pyonephrosis. NCCT reveald a stoen of 1.2
cm at the upper pole of right kidney. Patient underwent ESWl which failed to provide any solutions.
The stone was retrieved by renoscopy . Identify the stone shown in the pics after removal from the
patient:

a. Calcium Oxalate Monohydrate

b. Struvite

c. Matrix stone

d. Calcium oxalate dihydrate

Solution. c
# the stone shown in the image is Matrix stone.
#protein rich stone
#associated with recurrent infections
#ESWL resistant as it is too soft
#requires resnoscopic lithotripsy

Answer. c
214. 32 year old female presenting with painful , tender cord like vessel over the right inferior aspect
of the breast . All except are true for the condition :

a. trauma is most common cause

b. associated with superficial thrombophlebitis

c. surgical excision is the preferred management

d. may be associated with protein c deficiency

Solution. c
# this is mondors disease .
#superficial thrombophlebitis @ anterior chest wall @infra mammary region
#presents as painful tender cord like vessel
#nsaid’s are the 1st line management
#refractory cases requires surgical excision

Answer. c

215. Thenar flap is example of :

a. Transposition falp

b. Free flap

c. Interposition flap

d. Advancement flap

Solution. c
# interposition flap.
#this is a transposition flap which is interposed for regional reconstruction . Eg:
finger tip reconstruction by thenar flap

Answer. c

216. All except are inclusion criteria for surgery in AAA:

a. Cross sectional Diameter > 5.5cm

b. increase >1.5cm /year

c. increase >.5cm /6m

d. all are inclusion

Solution. b
# criteria for resection of abdominal aortic aneurysm is :
@ cross sectional diameter >5.5cm
@increase in diameter >1cm/year or >5mm /6 months

Answer. b
217. A 22-year old young male, college student, suffered a left knee injury while playing hockey. After
2 months, there was anterior laxity in full extension and it was normal at 90O flexion. What is the most
likely injured part?

a. Anteromedial bundle of anterior cruciate ligament

b. Posterolateral bundle of anterior cruciate ligament

c. Posterior cruciate ligament

d. Anterior part of medial meniscus

Solution. (b) Posterolateral bundle of anterior cruciate ligament


Ref: Peterson W, Zantop T. Level V Evidence: Partial Rupture of Anterior Cruciate Ligament.
Arthroscopy: The Journal of Arthroscopic and Related Surgery. 2006;22(11):1143-1145.
Sol:
- The anterior cruciate ligament (ACL) consists of two major fibre bundles namely the
anteromedial and posterolateral bundles.
- Although, disagreement exists among orthoscopic surgeons about the occurrence of the isolated
rupture of AM or PL bundles, there are reports of partial rupture of ACL in the literature.
- A potential reason for the controversy could be that; with the conventional MRI, isolated PL
ruptures are difficult to diagnose because of oblique course of bundle.
- Another reason could be that isolated ruptures of AM or PL bundles are difficult to diagnose
during arthroscopy.
- During arthroscopy an isolated PL bundle rupture can easily be missed when viewing from the
standard anterolateral portal.
- The AM bundle overlies the PL bundle and the PL bundle can only be seen by retraction of AM
bundle with a probe.
- When the knee is extended, the PL bundle is tight, and the AM bundle is moderately lax.
- As the knee is flexed the femoral attachment of ACL becomes horizontally oriented, causing the
AM bundle to tighten and the PL bundle to relax.
- As per the question, the patient has anterior laxity in full extension which is suggestive of PL
bundle rupture.
- Whereas the AM bundle is the primary restraint against anterior tibial translation in flexion, the
PL bundle tends to stabilize the knee near full extension, particularly against the rotatory loads.
- Isolated rupture of AM bundle has more effect on anterior drawer test than on the Lachmann
test whereas the converse is true for isolated rupture of PL bundle.

Answer. b
218. All of the following can be the result of carpal tunnel syndrome except :

a. Paralysis of opponens pollicis

b. Paralysis of flexor pollicis brevis

c. Paralysis of oblique head of adductor pollicis

d. Paralysis of abductor pollicis brevis

Solution. c) Paralysis of oblique head of adductor pollicis


Ref.: BD Chaurasia, Vol 1 Pg. 84.
Sol :
- Compression of median nerve in the carpal tunnel gives rise to sensory and motor symptoms in
the hand, which constitutes the carpal tunnel syndrome. There is pain in the distribution of the
median nerve.
- There may be wasting of the thenar muscles supplied by the median nerve – opponens pollicis,
superficial head of flexor pollicis brevis and abductor pollicis brevis.
- Adductor pollicis is supplied by the deep branch of ulnar nerve and is not affected in carpal
tunnel syndrome.

Answer. c

219. During replantation of an amputated limb which of the following is done first

a. Arterial repair

b. Venous repair

c. Fixation of the bone

d. Nerve anastomoses

Solution. (c) Fixation of the bone


Ref: Read the text below
Sol:
ORDER OF REPAIR
After all structures have been thoroughly cleansed, débrided, and identified, repair is begun. As
indicated in the discussion that follows, certain conditions or circumstances dictate a variation in
the order of repair. The following is our usual order of repair of damaged structures. Discussions
of digit, hand, and arm replantations are included.
1. Shorten and internally fix bone.
2. Repair extensor tendons.
3. Repair flexor tendons (2 and 3 may be reversed, or flexor tendon repair may be delayed).
4. Repair arteries.
5. Repair nerves.
6. Repair veins.
7. Close or cover wound.

Answer. c
220. An elderly female is on treatment for osteoporosis with alendronate for 7 years. She now presents
with complaints of hip pain. The best investigation for her would be –

a. X-ray

b. DEXA scan

c. Vitamin D levels

d. ALP levels

Solution. a) X-ray
Ref: The American Journal of Geriatric Pharmacotherapy, Volume 9pg 194–198.
Sol:
In the above case scenario patient is having pain & she is on alendronate since last 7 years, which
is highly suggestive of fragility fracture due to prolonged use of Bisphosphonates. X-ray would be
best investigation to see fractures.
- Despite the great success achieved by using bisphosphonates in osteoporotic patients during the
last two decades, there remains a group of patients who may develop femoral fragility fractures
secondary to Severely suppressed Bone Turnover (SSBT) caused by prolonged bisphosphonate
use, mainly with alendronate.
- The number of these cases is still relatively small but will probably increase in the future owing
to the large number of patients treated with these drugs.
- It is therefore important to reserve continuation of bisphosphonates therapy for more than five
years for selected cases.
- Furthermore, clinicians should be aware of the association between long-term bisphosphonates
therapy and femoral fragility fractures.
- In patients with early changes, such as prodromal hip/thigh pain and lateral cortical thickness,
stopping bisphosphonate therapy and prophylactic nailing should be considered.
- In patients with femoral fragility fractures, healing disturbances are not uncommon, and should
be anticipated and treated accordingly.
- If continuation of antiresorptive therapy is indicated despite the occurrence of femoral fragility
fractures, teriparatide represents a promising alternative to bisphosphonates because it reduces
microdamage accumulation caused by Severely suppressed Bone Turnover (SSBT).

Figure. Radiographs of the (A) right and (B) left hips of a patient who presented with bilateral,
sequential bisphosphonate-related subtrochanteric fractures. The transverse fracture pattern on
the lateral half of the femoral cortex both in the right and left hips raises the suspicion of a
possible preexisting linear crack involving the lateral half of the femoral cortex, which gave way
and progressed into an oblique fracture on the medial cortex. There is also significant cortical
hypertrophy of the femoral shaft, bilaterally below the fracture.

Answer. a
221. All are features of Paget’s disease except:

a. Defect in osteoclasts

b. Calcitonin is the drug of choice

c. Can cause deafness

d. Can cause osteosarcoma

Solution. B
Bisphosphonates are the drug of choice for Paget’s Disease of Bone.

Answer. b

222. Which of the following is not a feature of Colle’s fracture –

a. Volar tilt

b. Radial displacement

c. Dorsal displacement

d. Supination

Solution. (a)Volar tilt


Displacements seen in Colles Fracture :
D : Dorsal
I : Impaction
L : Lateral
S : Supination

Answer. a
223.
Name of this cast is :

a. PTB cast

b. Below knee cast

c. Above knee cast

d. Cylindrical cast

Solution. A
PTB CAST : Patellar Tendon Bearing Cast is used in Fracture Shaft of Tibia

Answer. a
224. Identify the type of SPINE shown in the Xray below :

a. Fish Mouth Spine

b. Rugger Jersey Spine

c. Picture Frame Spine

d. Bamboo Spine

Solution. A
Fish Mouth Spine is seen in Osteomalacia > Osteoporosis

Answer. a

225. Which of the following is the most commonly injured structure during laryngoscopy?

a. Lower central incisor

b. Larynx

c. Tongue

d. Upper central incisor

Solution. D
Upper central incisors are the most common injured structures during laryngoscopy .

Answer. d
226. Identify the capnogram shown below?

a. hyperventilation

b. rebreathing

c. hypoventilation

d. bronchospasm

Solution. C
- This is a capnogram of hypercapnia which is seen in hypoventilation or fever or sepsis.
- There is a gradual increase in the capnography due to accumulation of CO2 in the lungs

Answer. c

227. Which of the following is the LEAST appropriate technique for induction of general anesthesia in
a newborn for surgical repair of Tracheal Esophageal Fistula ( TEF)?

a. Awake tracheal intubation

b. Inhalation induction with spontaneous ventilation and tracheal intubation

c. Inhalation induction using positive-pressure bag and mask ventilation and tracheal intubation

d. Rapid-sequence IV induction and tracheal Intubation

Solution. (C)
- Anesthesia for patients with TEF can be safely induced with either an intravenous or volatile
anesthetic.
- However, positive-pressure bag and mask ventilation should be avoided because it will force gas
into the stomach, potentially making ventilation of the lungs more difficult.
- A frequently used technique to facilitate correct placement of the endotracheal tube is to
advance the tube into a bronchus.
- While listening over the stomach, slowly withdraw the tube until breath sounds are heard over
the stomach.
- Advance the tube until these sounds become diminished.
- Bronchoscopy is used by some anesthesiologists to make sure only one fistula is present and to
help position the endotracheal tube

Answer. c
228. Fade in response to tetanic stimulus is seen

a. True of nondepolarizing blockade only

b. True of phase I depolarizing blockade only

c. True of phase II depolarizing blockade only

d. True of nondepolarizing and phase II depolarizing Blockade

Solution. (D)
- The amplitude of the muscle response to sustained tetanic stimulation remains the same with phase I
depolarizing blockade, but it shows a marked fade with a phase II depolarizing blockade or a
nondepolarizing blockade.

Answer. d

229. Adrenal suppression

a. Ketamine

b. Diazepam

c. Etomidate

d. Propofol

Solution. C)
- Etomidate is unique among the intravenous induction agents because it can cause
adrenocortical suppression by inhibiting the conversion of cholesterol to cortisol.
- This can occur after a single induction dose and may persist for 4 to 8 hours.
- The clinical significance of this temporary adrenocortical suppression is unclear.
- However, in the ICU with prolonged sedation, clinical adrenal insufficiency may develop (i.e.,
hypotension, hyponatremia, and hyperkalemia).
- Here corticosteroids should be administered in stress doses (e.g., cortisol 100 mg/day)

Answer. c
230. A 35-year-old patient with a history of grand mal seizures is anesthetized for thyroid surgery
under general anesthesia consisting of midazolam and an infusion of propofol and remifentanil for
analgesia. The patient takes phenytoin for control of seizures. After surgery the patient is transported
intubated to the recovery room where he is arousable, but not breathing. The most reasonable course
of action would be

a. Administer naloxone

b. Administer flumazenil

c. Administer naloxone and flumazenil

d. Ventilate by hand

Solution. D)
- Patients anesthetized with total intravenous anesthesia (TIVA), in this case consisting of
midazolam, remifentanil, and propofol, sometimes require a few minutes to resume breathing
after the infusions are stopped.
- Although it may seem appropriate to reverse this patient and avoid the need for hand
ventilation, reversing benzodiazepines (midazolam) with flumazenil may precipitate seizures in
epileptic patients, and, because remifentanil has such a short elimination half-life (<6 minutes),
reversal with naloxone is not necessary.
- Also, muscle weakness must be ruled out if a muscle relaxant has been used, and normocapnia
should be assured given that hyperventilation may reduce the arterial CO2 below the apnoeic
threshold

Answer. d

231. Each of the following results in a reduction of the incidence of postoperative vomiting (POV) in
children undergoing strabismus surgery EXCEPT

a. adequate hydration

b. Dexamethasone

c. Ondansetron

d. Anticholinergics (atropine or glycopyrrolate)

Solution. D)
- Prophylaxis for POV is recommended for patients undergoing strabismus surgery, because
untreated, the incidence is 40% to 90% of patients.
- No benefit was demonstrated with the use of anticholinergic medications or with gastric content
evacuation before emergence from anesthesia.
- IV hydration is very important.
- Decreasing or avoiding narcotic analgesics has also been effective.
- Avoiding the maintenance use of nitrous oxide remains controversial

Answer. d
232. Which of the following is TRUE regarding intravenous regional anesthesia (Bier block)?

a. Useful for postoperative pain in extremity surgery

b. Can be used for extremity surgeries lasting 2 to 3 hours

c. Bupivacaine is the drug of choice for prolonged blocks

d. Lidocaine is most commonly used

Solution. D)
- Intravenous regional anesthesia (IVRA, or Bier blocks after August Bier, who first described the
technique) is simple to perform and is usually done only on an upper extremity.
- A small 20- or 22-gauge IV catheter is placed in the extremity to be blocked, then the limb is
raised and an Esmarch bandage is wrapped around the extremity to remove as much blood from
the limb as possible, followed by the inflation of a tourniquet to 250 to 300 mm Hg, or 2.5 times
the patient’s systolic pressure, and injection of a local anesthetic into the limb.
- Postoperative analgesia is lost once the tourniquet is deflated and the local anesthetic diffuses
from the nerves.
- Tourniquet times less than 60 to 90 minutes are used to prevent pain and nerve damage from
the tourniquet.
- Prilocaine and Lidocaine 0.5% at a dose of 1.5 to 3 mg/kg is the most commonly administered
local anesthetic because of its relative safety and effectiveness.
- Bupivacaine is not recommended for Bier blocks because of reports of cardiovascular toxicity
and death that have occurred after the tourniquet was released

Answer. d

233. An ideal therapeutic radioisotope is

a. Strong Beta; Strong Gamma emitter

b. Strong Beta; Low Gamma emitter

c. Low Beta; Low Gamma emitter

d. Low Beta; Strong Gamma emitter

Solution. b) Strong beta, low gamma emitter An ideal therapeutic radioisotope is a strong beta
emitter with just enough gamma to enable imaging (e.g. lutetium-177 ).

Answer. b
234. Patient presents with acute abdomen to surgical emergency. CT abdomen shows:

a. Liver abscess

b. Acute pyelonephritis

c. Pneumoperitoneum

d. Acute pancreatitis

Solution. c) pneumoperitoneum
Abdominal radiograph
Free gas within the peritoneal cavity can be detected on an abdominal radiograph. These signs
can be further divided by anatomical compartments in relation to the pneumoperitoneum:
bowel related signs
- double wall sign (also known as Rigler's sign or bas-relief sign)
- telltale triangle sign (also known as triangle sign)
peritoneal ligament related signs
- football sign
- falciform ligament sign
- lateral umbilical ligament sign (also known as inverted "V" sign)
- urachus sign
right upper quadrant signs
- lucent liver sign
- hepatic edge sign
- fissure for ligamentum teres sign
- Morison's pouch sign (doge cap sign)
- cupola sign

Answer. c
235. Brachytherapy: what is not true?

a. Improve conformality by 3-D techniques

b. Useful in carcinoma esophagus

c. Same proportion of radiation as teletherapy in localised area

d. More effective in Ca Cervix

Solution. c)
Brachytherapy
- Delivers higher amount of radiation in a localised area, with a better local control.
- Large tumours are usually unsuited for use
- Conformality of dose-delivery to target tissues is a major problem, to overcome which, newer 3-D
conformal techniques are being developed (like 3D-IBT, a type of 3-dimensional interstitial
brachytherapy), which have produced good results in cancer cervix.
- Preferred in cancer cervix, uterus, vagina as well as breast, brain and esophageal cancers
- Can be given intracavitary, interstitial or surface route

Answer. c

236. A 22-year old man has had a swollen wrist for several months. Diagnosis please.

a. Osteosarcoma

b. Osteoclastoma

c. Aneurysmal Bone Cyst

d. Simple Bone cyst

Solution. b)
This lesion has a benign radiographic appearance. Note that, while the distal radius is expanded,
the cortex of the bone is intact. The margin of the tumor is well demarcated. This is a giant cell
tumor. These are usually benign although approximately ten percent can be malignant. The key
radiographic feature is the extension of the lesion to the articular surface.

Answer. b
237. 25 years old man is involved in an automobile accident and slams his head into a concrete wall of
a bridge. His CT scan is shown to you. Blood leaking the ruptured artery enters which of the following
spaces?

a. Subarachnoid space

b. Subdural space

c. Epidural space

d. Subpial space

Solution. c)
This is EDH, which is biconvex and limited by skull sutures. Due to rupture of middle meningeal
artery.

Answer. c
238. Identify the area marked in the brain

a. Putamen

b. Hippocampus

c. Amyglada

d. Globus pallidus

Solution. D

Answer. d
239. A 31-year-old woman develops mild acute pancreatitis and is managed conservatively. It is her
third episode of pancreatitis but there is no history of excess alcohol consumption and an abdominal
ultrasound is normal. Magnetic resonance cholangiopancreatography (MRCP) is performed and is
reported as showing evidence of pancreas divisum. Which one of the following findings is likely to have
been present on MRCP?

a. An accessory pancreatic duct passing around the duodenum

b. The common bile duct draining into the minor papilla

c. The dorsal pancreatic duct (duct of Santorini) draining into the minor papilla

d. The ventral pancreatic duct draining into the minor papilla

Solution. C
It appears that relative stenosis of the cranially sited minor papilla results in increased risk of
pancreatitis in these patients.

Answer. c

240. CT Angiography in this patient in emergency for breathlessness shows:

a. Aortic Aneurysm

b. Aortic Dissection

c. Pulmonary embolism

d. SVC obstruction

Solution. c) Pulmonary embolism


- Pulmonary embolism is a common cause of morbidity and mortality in the ICU patient.
- The chest radiograph is of limited value.
i. It may be normal or reveal non-specific atelectasis.
ii. A peripheral area of more or less wedge-shaped consolidation may indicate associated
infarction (the so-called ‘Hampton’s hump’).
iii. Regional oligaema with sharp cut-off of pulmonary arteries may be seen (the Westermark
sign).
- Traditional investigations have included ventilation perfusion scintigraphy and pulmonary
angiography, but CT pulmonary angiography (CTPA) has now become the preferred technique for
confirming or excluding the presence of pulmonary embolism

Answer. c
241. Not included in antenatal screening

a. AFP

b. HCG

c. AMH

d. INHIBIN

Solution. C
AMH is a test of ovarian reserve and is not used in antenatal screening. HCG is used in both first
and second trimester serum screening. While AFP and inhibin A are used in second trimester
serum screening

Answer. c

242. Regarding Second generation endometrial ablation what is true

a. Ball rolling among second generation

b. Risk of endometrial cancer increased

c. Risk of pregnancy is there but minimum

d. Done in OT under GA

Solution. C
•Second generation endometrial ablation are non hysteroscopic methods and are done in out
patient setting with minimal anaesthesia. Ball rolling is a first generation method which are done
hysteroscopically. The second generation are less time consuming equally effective and need less
training and they cause global destruction of the endometrium. These methods do not increase
risk of endometrial cancer but they make assessment of endometrium difficult hence should not
be done when there is risk of cancer or when there is hyperplasia
•There is minimal risk of pregnancy with these methods and as they destroy endometrium hence
should be done only when family is complete

Answer. c
243. Which of the following is not incorrect about the image shown below

a. Recurrence risk after previous 1 such baby is 10%

b. Is associated with preterm labour

c. The female should be given 4mg folic acid before next conception

d. More common in male sex

Solution. C
This is an image of anencephaly and the recurrence risk after previous one is 4% while after 2
previous babies it is 10%. Since there is absence of fetal HP axis they g in to post term labour. It
is more common in female fetus. The woman should be given 4mg of folic acid to prevent
recurrence at least 3 moths before pregnancy and continued throughout first trimester

Answer. c
244. A 36-year-old G0 who has been epileptic for many years is contemplating pregnancy. She wants to
stop taking her phenytoin because she is concerned about the adverse effects that the medication may
have on her unborn fetus. She has not had a seizure in the past 5 years. Which of the following is the
most appropriate statement to make to the patient?

a. Babies born to epileptic mothers have an increased risk of structural anomalies even in the
absence of anticonvulsant medications.

b. She should discontinue her phenytoin because it is associated with a 1% to 2% risk of spina
bifida.

c. Vitamin C supplementation reduces the risk of congenital anomalies in fetuses of epileptic


women taking anticonvulsants.

d. The most frequently reported congenital anomalies in fetuses of epileptic women are limb
defects.

Solution. A
Offspring of women with epilepsy have 2 to 3 times the risk of congenital anomalies even in the
absence of anticonvulsant medications, because seizures cause a transient reduction in uterine
blood flow and fetal oxygenation. When anticonvulsant medications are used, pregnant women
have an even greater risk of congenital malformations. It is recommended that women undergo a
trial of being weaned off their medications prior to becoming pregnant. If antiseizure medications
must be used, mono-therapy is preferred to minimize the risk to the fetus, since the incidence of
fetal anomalies increases as additional anticonvulsants are consumed. Many anticonvulsants have
been found to impair folate metabolism, and folate supplementation in pregnancy has been
associated with a decreased incidence of congenital anomalies in epileptic women taking
antiseizure medications. Fetal exposure to valproic acid has been associated with a 1% to 2% risk
of spina bifida.

Answer. a

245. Delivery at 32-34 weeks termed as

a. Early preterm

b. Late preterm

c. Moderate preterm

d. Near term

Solution. C
•Babies born netwen 34-36+6 weeks are called late preterm
•Between 32-36 moderately preterm
•Less than 32 weeks is very preterm
•Babies born between 37-38+6 is early term between 39-40+6 is term
•While 41-41+6 late term and beyond 42 weeks is post term

Answer. c
246. A patient presents to your office for her first prenatal visit. By her last menstrual period she is 11
weeks pregnant. At this visit you observe that her uterus is palpable midway between the pubic
symphysis and the umbilicus. No fetal heart tones are audible with the Doppler stethoscope. Which of
the following is the best next step in the management of this patient?

a. Reassure her that fetal heart tones are not yet audible with the Doppler stethoscope at this
gestational age.

b. Schedule genetic amniocentesis right away because of her advanced maternal age.

c. Schedule her for a dilatation and curettage.

d. Schedule an ultrasound as soon as possible to determine the gestational age and viability of the
fetus.

Solution. D
At 11 weeks of gestation, the uterus is still within the pelvis and should not be palpable above the
symphysis pubis. A uterus that is palpable midway between the symphysis pubis and the
umbilicus is 14 to 16 weeks in size. The fetal heart tones are audible in most patients at 10 weeks.
If no fetal heart tones are audible by Doppler auscultation and the patient is 10 weeks or more, an
ultrasound of the pregnancy should be ordered. Molar pregnancy, twin gestation, incorrect dates,
and uterine fibroids are all possible diagnoses when the uterus is large for dates; therefore,
ultrasonography is the first step in the evaluation of size/date discrepancy. Although molar
pregnancy is an indication for dilatation and curettage, the procedure is not indicated before
evaluation of the patient with ultrasonography. This patient is of advanced maternal age (>35
years of age at the time of delivery), however, genetic amniocentesis should not be performed
without first knowing the gestational age and viability of the pregnancy

Answer. d

247. Which of the following statements is true regarding emergency contraception?

a. If an established pregnancy is present use of Plan B will cause an abortion.

b. Mifepristone is less effective than the Yuzpe method.

c. The emergency contraceptive, Plan B, requires a prescription.

d. The major mechanism of action of emergency hormone contraceptives is inhibition or delay of


ovulation.

Solution. D
•plan B is LNG tab 1.5mg single dose or 0.75mg two divided doses 12 hrs apart
•Emergency contraceptives are not abortifacients they done act after implantation
•Yuzpe regimen is high dose OCP 100mcg EE and 0.5mg LNG taken twice 12 hrs apart and
among hormonal methods it is least effective with maximum side effects
•Emergency contraception are over the counter drugs and do not need prescription
•Hormonal EC act by delaying ovulation

Answer. d
248. A 22-year-old G1P0 at 28 weeks gestation by LMP presents complaining of decreased fetal
movement. On the fetal monitor there are no contractions. The fetal heart rate is 150 beats per minute
and reactive. There are no decelerations in the fetal heart tracing. on ultrasound The following is seen.
This patient is most at risk for which of the following?

a. Premature rupture of the membranes

b. Fetal exsanguination after rupture of the membranes

c. Torsion of the umbilical cord caused by velamentous insertion of the umbilical cord

d. Placenta accrete

Solution. B
This patient has a Vasa Previa. When fetal vessels cross the internal os (vasa previa), rupture of
membranes may be accompanied by rupture of a fetal vessel leading to fetal exsanguination. Vasa
previa does not increase the risk for placenta accreta or amniotic fluid embolism. With
velamentous insertion of the cord, the umbilical vessels separate in the membranes at a distance
from the placental margin which they reach surrounded only by amnion. Such insertion occurs in
about 1% of singleton gestations but is quite common in multiple pregnancies. Fetal
malformations are more common with velamentous insertion of the umbilical cord. An increased
risk of premature rupture of membranes and of torsion of the umbilical cord has not been
described in association with velamentous insertion of the cord.

Answer. b
249. Five patients present for contraceptive counselling, each requesting that an IUD be inserted.
Which of the following is a contraindication to the use of an IUD?

a. Abnormalities of the uterus resulting in distortion of the cavity

b. Chorioamnionitis during pregnancy 6 months ago

c. History of loop electrocautery excision procedure of the cervix

d. History of recurrent candida vulvitis

Solution. A
A previous pregnancy with an IUD is not a contraindication to the use of an IUD. The risk of
another pregnancy with the IUD in place is not increased. Previous cervical surgery in the face of
a normal Pap smear and no cervical stenosis is not a contraindication to IUD use. The
manufacturers list the following contraindications to the use of an IUD: (1) pregnancy or
suspicion of pregnancy; (2) abnormalities of the uterus resulting in distortion of the uterine
cavity; (3) acute pelvic inflammatory disease (PID); (4) untreated acute cervicitis or vagnitis,
including bacterial vaginosis, until the infection is controlled; (5) postpartum endometritis or
septic abortion within the past 3 months; (6) genital bleeding of unknown etiology; (7) known or
suspected uterine or cervical neoplasia, or unresolved abnormal cytological smear

Answer. a

250. A 16-year-old primigravida presents to your office at 38 weeks gestation. Her first trimester blood
pressure was 100/72. Today it is 156/96 mm Hg and she has 2+ proteinuria on a clean catch specimen
of urine. She has significant swelling of her face and extremities. She denies having contractions. Her
cervix is closed and uneffaced. The baby is breech by bedside ultrasonography. Which of the following
is the best next step in the management of this patient?

a. Send her to labor and delivery for a BPP.

b. Admit her to the hospital for enforced bed rest and diuretic therapy to improve her swelling
and blood pressure.

c. Admit her to the hospital for induction of labor.

d. Admit her to the hospital for cesarean delivery.

Answer. d
251. A 23-year-old G1P0 at 42 weeks is undergoing induction of labor. She is receiving intravenous
oxytocin. She complains that her contractions are very painful and seem to be continuous. The following is
the CTG graph for her. What is the finding suggestive of

a. Placental insufficiency

b. Hyperstimulation

c. Cord compression

d. Fetal hypoxia

Solution. B
Tracing shows a classic hyperstimulation pattern, with a tonic contraction lasting several minutes
with distinctly raised intrauterine pressure and a consequent fall in fetal heart rate. Despite the
increased uterine pressure, there remains good beat-to-beat variability, which suggests that the
fetus is withstanding the stress

Answer. b
252. A 62 year old woman presents with pressure in the pelvis On performance of a physical
examination, you note the following . Based on the physical examination, which of the following is the
most likely diagnosis?

a. Normal examination

b. First-degree uterine prolapse

c. Second-degree uterine prolapse

d. Third-degree uterine prolapse

Solution. C
The degree or severity of pelvic relaxation is rated on a scale of 1 to 3, based on the des cent of
the organ or structure involved. First-degree prolapse involves descent limited to the upper two-
thirds of the vagina. Second-degree prolapse is present when the structure is at the vaginal
introitus. In cases of third-degree prolapse, the structure is outside the vagina. Total procidentia
is when entire uterus with the fundus lies outside

Answer. c
253. A 76-year-old woman presents for evaluation of urinary incontinence.. After complete evaluation,
you determine that the patient has genuine stress urinary incontinence. On physical examination, she
has a hypermobile urethra, but there is no cystocele or rectocele. Which of the following surgical
procedures should you recommend to this patient?

a. Anterior and posterior colporrhaphy

b. A midurethral sling procedure

c. Abdominal sacral colpopexy

d. Burch procedure

Solution. B
A midurethral sling procedure such as the transobturator tape or transvaginal tape placement
would be the most appropriate surgical treatment for stress urinary incontinence in this patient..
The Burch procedure suspends the bladder neck to Cooper ligament of the pubic bone using an
abdominal approach. Anterior and posterior colporrhaphy are procedures used to correct
cystoceles and rectoceles and are not indicated in this patient. Sacral colpopexy is a procedure to
repair prolapse of the vagina by suspending the vaginal vault from the sacrum. The midurethral
sling procedures are minimally invasive requiring only small vaginal and skin incisions.
Additionally they may be performed as an outpatient procedure

Answer. b

254. Which of the following is done for cervical cancer screening

<21 years>a.

b. > 30 years every 5 yearly

c. 21-65 years

d. >65 years

Solution. c
•Cervical cancer screening as per international guidelines is to be done between 21-65 years of
age
•Between 21-25 only PAP smear is to used and is done every 3 yearly beyond 25 years we can
either use PAPS alone every 3 yearly or use cotest every 5 yearly
•If screening Iast decade is normal then we stop screening at 65 years

Answer. c
255. Which of the following statement about the pathology shown below is not true

a. Secondary amenorrhea

b. IOC is ultrasound

c. Cause is vigorous curettage

d. Post operatively treat with estrogen

Solution. B
This is an image of ashermans syndrome. The investigation of choice for it is hysteroscopy it is
both diagnostic and the therapeutic. To prevent recurrence after hysteroscopic adhesiolysis the
patient is treated with high dose estrogen for 1 cycle to prevent recurrence

Answer. b

256. An infertile couple presents to you for evaluation. A semen analysis from the husband is ordered.
The sample of 2.5 cc contains 25 million sperm per mL; 15% of the sperm show normal morphology;
20% of the sperm show progressive forward mobility. You should tell the couple which of the following?

a. The sample is normal and should not be a factor in the couple’s infertility.

b. The sample is abnormal because the percentage of sperm with normal morphology is too low.

c. The sample is abnormal because of an inadequate number of sperm per milliliter.

d. The sample is abnormal owing to a low percentage of forwardly mobile sperm

Solution. D
•the minimum requirement of semen parameters as per WHO are
•volume 1.5ml
•total sperm count 39milliom/ejaculate
•sperm concentration 15million/ml
•morphology 4%
•total motility 40% while forward motility 32%

Answer. d
257. A 45-year-old woman who had two normal pregnancies 15 and 18 years ago presents with the
complaint of amenorrhea for 7 months. She expresses the desire to become pregnant again. After
exclusion of pregnancy, which of the following tests is next indicated in the evaluation of this patient’s
amenorrhea?

a. Hysterosalpingogram

b. Endometrial biopsy

c. Thyroid function tests

d. FSH level

Solution. D
She is in the perimenopausal age group and if the woman wants to conceive then we must
evaluate her ovarian reserve for pregnancy. The most commonly used test of ovarian reserve is
serum FSH levels. Levels less than 10IU is normal while 10-15 is borderline reserve and more
than 15 is poor ovarian reserve . best test of ovarian reserve is AMH levels .

Answer. d

258. A 42 year old lady with complains of dysmenorrhea dyspareunia and chronic pelvic pain
undergoes hysterectomy. The following is the image what is the diagnosis

a. Endometriosis

b. Fibroid uterus

c. Adenomyosis

d. Ectopic pregnancy

Solution. c
•This is a diffusely enlarged uterus and looks globular and along with the complains of
dysmenorrhea and chronic pelvic pain and dyspareunia the diagnosis is Adenomyosis.
•In fibroid the uterus is assymetrically enlarged and in endometriosis the uterus is not enlarged in
size
•In ectopic pregnancy we don’t do a hysterctomy and will present with bleeding not as
dysmenorrhea and dyspareunia

Answer. c
259. A 29-year-old G3P2 presents to the emergency with complaints of abdominal discomfort for 1
weeks. Her blood pressure is 120/70 mm Hg, pulse 90 beats per minute, temperature 36.9°C,
respiratory rate 18 breaths per minute. A pregnancy test is positive and an ultrasound of the abdomen
and pelvis reveals a viable 16-week gestation located behind a normal-appearing uterus. Both ovaries
appear normal. No free fluid is noted. Which of the following is the most likely cause of these findings?

a. Ectopic ovarian tissue

b. Fistula between the peritoneum and uterine cavity

c. Primary peritoneal implantation of the fertilized ovum

d. Tubal abortion

Solution. D
Almost all cases of abdominal pregnancy follow early rupture or abortion of a tubal pregnancy.
Women with abdominal pregnancy are likely to be uncomfortable, but with vague gastrointestinal
symptoms such as nausea, vomiting, flatulence, constipation, and diarrhea. Fetal survival is
precarious with a perinatal loss of 75%. Fetal malformations and deformities, such as craniofacial
asymmetry, limb deficiencies, and joint abnormalities, are present in 20% of fetuses. Expectant
management carries the risk of sudden life threatening haemorrhage and is rarely done if the
diagnosis of abdominal pregnancy is made. Surgery is the usual treatment of abdominal
pregnancy, but massive hemorrhage may ensue with separation and removal of the placenta. In
general, the fetus should be delivered, the cord severed close to the placenta, and the abdomen
closed.

Answer. d
260. A 32-year-old G5P1 presents for her first prenatal visit. A complete obstetrical, and medical
history and physical examination is done. Which of the following would be an indication for elective
cerclage placement?

a. Three spontaneous first-trimester abortions

b. two second-trimester pregnancy losses without evidence of labor

c. History of loop electrosurgical excision procedure for cervical dysplasia

d. Cervical length of 35 mm by ultrasound at 18 weeks

Solution. B
The diagnosis of cervical insufficiency or incompetence is based on the presence of painless
cervical dilation with a history of pregnancy loss in the second trimester or early-third-trimester
preterm delivery. A patient with a history of three or more mid trimester pregnancy losses or
early preterm deliveries is a candidate for a circlage. Circlage is not indicated for the prevention
of first-trimester losses. Circlage has not been shown to improve the preterm delivery rate or
neonatal outcome in twin gestations. A simple punch biopsy or loop electrosurgical excision
procedure of the cervix is unlikely to disrupt functional structure of the cervix and prophylactic
cerclage is not warranted. Serial transvaginal ultrasound evaluation of cervical length can be
considered in women with a history of second and early-third-trimester deliveries. A cervical
length less than 25 mm or funneling of more than 25% or both is associated with an increased
risk of preterm delivery

Answer. b

261. A woman at 41 weeks of gestation is very sure of her LMP has no uterine contractions no
effacement and no dilatation. Which of the following is not to be used

a. Intracervical foley catheter

b. PG tablet

c. PG injection

d. PG gel

Solution. C
PFF2alpha or carboprost is not used for induction of labour or cervical ripening in a term
pregnancy although it can be used for second trimester induced abortions. It is also used in
management of PPH, the dose is 0.25mg and can be repeated up to 8 times every 15-30 min

Answer. c
262. An in-depth sonogram on a 30-year-old G1 at 28 weeks with a twin gestation indicates that the
fetuses are both male, and the chorionicity is diamniotic and monochorionic. Twin B is noted to have
oligohydramnios and to be much smaller than twin A. Which of the following would be a finding most
likely associated with twin A?

a. Congestive heart failure

b. Anemia

c. Hypovolemia

d. Low amniotic fluid levels

Solution. A
•In twin gestations where monochorionic placentas exist, twin-to-twin transfusion syndrome can
occur. In this syndrome, there are vascular communications or anastomoses between the twins.
There is blood flow or transfusion from one twin to another. The donor twin becomes anaemic and
may suffer growth retardation and oligohydramnios. The recipient twin may develop hydramnios,
hypervolemia, hypertension, polycythemia, and congestive heart failure

Answer. a

263. A mother brings her 14-year-old daughter in to the office for consultation. She says her daughter
should have started her period by now. She is also concerned that she is shorter than her friends. On
physical examination, the girl is 4 ft 10 in tall. She shows evidence of breast development at Tanner
stage 2. She has no axillary or pubic hair. Educating the mother and daughter, your best advice is to
tell them which of the following?

a. The daughter will start her period when her breasts reach Tanner stage 5.

b. The daughter will start her period, then have her peak height .

c. The daughter’s period should start within 1 to 2 years since she has just started developing
breast buds.

d. The daughter will have her height peak, then pubic hair will develop, heralding the onset of
menstruation.

Solution. C
Significant emotional concerns develop when puberty is delayed. By definition, if breast
development has not begun by age 13, delayed puberty should be suspected. Menarche usually
follows about 1 to 2 years after the beginning of breast development; if menarche is delayed
beyond age 15, delayed puberty should be investigated. Appropriate laboratory tests include
circulating pituitary and steroid hormone levels, karyotypic analysis, and central nervous system
(CNS) imaging when indicated. An FSH value greater than 40 mIU/mL defines hypergonadotropic
hypogonadism as a cause of delayed pubertal maturation. Hypergonadotropic hypogonadism is
seen in girls with gonadal dysgenesis, such as occurring with Turner syndrome. S

Answer. c
264. What percentage of these tumours are bilateral

a. Less than 1%

b. 2% to 3%

c. 10%

d. 50%

Solution. C
Benign cystic teratomas (dermoids) are the most common germ cell tumors and account for about
20% to 25% of all ovarian neoplasms. They occur primarily during the reproductive years, but
may also occur in postmenopausal women and in children. Dermoids are usually unilateral, but
10% are bilateral. Usually the tumours are asymptomatic, but they can cause severe pain if there
is torsion or if the sebaceous material perforates, spills, and creates a reactive peritonitis.

Answer. c

265. A 44-year-old G6P3215 presents for her well-woman examination. She tells you that all of her
deliveries were vaginal and that her largest child weighed 2900 g at birth. How many full-term
pregnancies did this patient have?

a. 2

b. 3

c. 5

d. 6

Solution. B
When taking an obstetric history on a patient, you must indicate the number of pregnancies
(gravidity) and the outcome of each of these pregnancies (parity). More specifically, the parity is
further subclassified into number of term deliveries, preterm deliveries, abortions (spontaneous
or induced) or ectopics, and number of living children. Since this patient is a G6P3215, she has
been pregnant six times and has had three term deliveries, two preterm deliveries, one abortion,
and has five living children

Answer. b
266. A 34-year-old G3P2 delivers a baby by spontaneous vaginal delivery. At first glance you notice
female genitalia, but on closer examination the genitalia are ambiguous. Which of the following is the
next step in the evaluation of this infant?

a. Chromosomal analysis

b. Evaluation at 1 month of age

c. Pelvic ultrasound

d. Thorough physical examination

Solution. D
Ambiguous genitalia at birth is a medical emergency, not only for psychological reasons for the
parents but also because hirsute female infants with congenital adrenal hyperplasia (CAH) may
die if undiagnosed. CAH is an autosomally inherited disease of adrenal failure that causes
hyponatremia and hyperkalemia because of lack of mineralocorticoids. A thorough physical
examination is the best initial evaluation. While it will not give the definitive diagnosis of the sex,
it can provide clues. Are the gonads palpable in the inguinal canal? Are the labia fused? Is there a
vagina or pouch? Is there hyper- or hypotension, or signs of dehydration. Karyotype, electrolyte
analysis, blood or urine assays for progesterone, 17α-hydroxyprogesterone, and serum androgens
such as dehydroepiandrosterone sulfate are essential to the workup. Pelvic ultrasound or MRI can
detect ovaries or undescended testes, but that is not the first step in management.

Answer. d
267. A 20-year-old G1 at 41 weeks has been pushing for 2½; hours. The fetal head is at the introitus
and beginning to crown. It is necessary to cut an episiotomy. The tear extends and the following is the
image, how would you classify this

a. first degree

b. Second-degree

c. Third-degree

d. Fourth-degree

Solution. C
first-degree tear involves the vaginal mucosa or perineal skin, but not the underlying tissue. In a
second-degree episiotomy, the underlying subcutaneous tissue is also involved, but not the rectal
sphincter or rectal mucosa. In a third-degree tear, the rectal sphincter is affected. A fourth-
degree episiotomy involves a tear that extends into the rectal mucosa

Answer. c

268. All the following vaccines are safe in pregnancy except

a. Diphtheria

b. Rabies

c. Influenza

d. Varicella

Solution. D
•MMR, HPV , varicella vaccines are not to be given in pregnancy
•While yellow fever can be given if the woman is travelling to endemic area, cholera and OPV can
be given if there is an outbreak and rabies can be given in case of dog bite

Answer. d
269. What instrument is shown below

a. Sponge holding forceps

b. Ovum forceps

c. Punch biopsy forceps

d. Babcocks

Solution. B
It has spoon shaped end and there is no lock in the instrument

Answer. b

270. Post menopausal hormone therapy is not indicated in

a. Hot flashes

b. CVD

c. Osteoporosis

d. Vaginal dryness

Solution. B
The womens health initiative reported an increase in events of coronary heart disease. For
younger postmenopausal women that is between 50-59 years and were less than 10 years since
menopause did not have excess risk or possibly had a reduction in risk. Prevention of CHD is not
an indication for initiation of postmenopausal HRT

Answer. b
271. A new born infant is being evaluated for ambiguous genitalia. While awaiting results of
chromosome analysis, the infant develops hyponatremia, hypoglycemia, and low blood pressure. What
is the most likely diagnosis?

a. 21- hydroxylase deficiency

b. Severe hypospadius

c. 3-beta-hydroxysteroid dehydrogenase deficiency

d. 11- beta – hydroxylase deficiency

Solution. (A) -21- hydroxylase deficiency The infant described in this scenario most likely has
congential adrenal hyperplasia of which more than 90% of cases are caused by 21- hydroxylase
deficiency. Ambiguous genitalia in association with congenital adrenal hyperplasia usually
indicates a female pseudoher- maphrodite (46XX karyotype with ambiguous external genitalia due
to excess androgens). This disorder is associated with both cortisol and aldosterone deficiency of
which the classic signs and symptoms are progressive weight loss and vomiting , hyponatremia,
hyperkalemia, and hypoglycemia.

Answer. a

272. Hydrocephalus, chorioretinitis, and diffuse cerebral calcifications are present in a new born male.
Of the following pets, which is most likely to be the source of this zoonotic congenital infection?

a. Dog

b. Cat

c. Horse

d. Rabbit

Solution. (B) Cat Cats are the definitive host for toxoplasma gondii, an intracellular parasite that
can be transmitted from mother to fetus through the placenta. Typically this is the result of a
primary infection in the mother. the most severely infected fetuses aquire the infection during the
first trimester of pregnancy .

Answer. b
273. Prior to the new born screening program in your state, you are evaluating an infant with a
skeletal survey due to concerns of trauma. Which of the following roentgen graphic finding in a
newborn infant is most suggestive of undiagnosed hypothyroidism?

a. Epiphyseal Dysgenesis

b. Absence of ossification of the hamate bone

c. Prominent thymic shadow

d. Osteoporosis

Solution. (a) Epiphyseal Dysgenesis


epiphyseal dysgenesis the development of multiple foci of ossification, occurs in patients with
hypothyroidism that are not treated or are inadequately treated . The development of ossification
centers is also rearted in hypothyroidism. About 60% of infants with congenital hypothyroidism
have x ray changes, consisent with delayed osseous development, present at birth. The
ossification center of the hamate is not normally present at birth , thus x ray examination of the
wrist is of no value in the newborn period .

Answer. a

274. A 5 day old appropriate for gestational age male presents with tachypnea, poor feeding, and
lethargy. On examination the neonate appears in shock with hypotension, pallor, and poor capillary
refill. Among the following, which is the most likely diagnosis?

a. Tetrology of Fallot

b. Tricuspid atresia

c. Transposition of the great vessels

d. Hypoplastic left heart syndrome.

Solution. (D) Hypoplastic left heart syndrome. Hypoplastic left heart syndrome typically presents
within the first few days to weeks of life with tachypnea, lethargy, and poor feeding.

Answer. d
275. A worried mother brings her 2 years old to the pediatrician for a well child check. She is
concerned because the child still has only 50 words in his Vocabulary. Which of the following best
described the characteristic age of a toddler who speaks 50 words?

a. 12 months

b. 15 month

c. 18 months

d. 24 months

Solution. D) 24 months Following the realization that words can stand for things , the child’s
vocabulary balloons from about 10-15 words at 18 months, there is a dramatic increase in
expressive and receptive vocabulary and by the end of the second year, quantum leap occurs in
language development , such that most children presenting for their 2 year-old checkup are
reported to say at least 50 words.

Answer. d

276. A 12 year old male is brought to your office for concerns regarding excessive drinking. The family
states that the excessive drinking is also causing him to urinate frequently . His weight has decresed 6
kg in the past 5 months . His current medications include albuterol as needed as well as lithium. A
urine analysis shows no glucose, ketones, or evidence of a urinary tract infection. The specific gravity
is less than . Further workup shows the serum sodium to be 163 meq/L with a normal potassium and
glucose. BUN is mildly elevated . Urine osmolality is less than serum osmolality. The urine
concentrates only minimally when parenteral DDAVP is administered. What parenteral DDAVP is
administered. What is the most helpful treatment for the patient’s condition?

a. Fludrocortisone

b. Fluid restriction

c. Intranasal DDAVP

d. Hydrochlorothiazide

Solution. D) Hydrochlorothiazide
The patient is presenting with diabetes insipidus(DI). This condition can be divided into two
forms: central or nephrogenic . Central DI is due to lack of antidiuretic hormone from the
pituitary gland and responds to DDAVP. Nephrogenic DI is due to renal unresponsiveness to
antidiuretic hormone . As this patient’s urine did not concentrate upon administration of DDAVP,
nephrogenic DI is the diagnosis . Nephrogenic DI can be treated with hydrochlorothiazide .
Although it may seem paradoxical to trat polyuria with a diuretic , depletion of sodium ( due to
diuretic therapy) results in increased reabsorption of sodium and water in the proximal tubule .
This results in a decreased in the water , lost due to the defect of ADH action, on the collecting
ducts. Nephrogenic DI can be congenital or acquired. The congential form is X linked and
presents in infancy in males.

Answer. d
277. A 13 month old boy presents with 5 days of fever to 103◦ F. His temperature is 102.8◦ F, HR 160
beats per minute, and RR 36 times per minute. On examination, he is found to be irritable with
markedly injected conjunctiva, a strawberry tounge, and red, cracked lips. A 2 cm lymph node is
present in the left aanterior cervical chain. There is no meningismus. His lungs are clear and he is
tachycardic. A diffuse erythematous blanching rash is present on his chest and extremities. No
desquamation of the fingertips is noted.
Among the following, which diagnostic test is most important in establishing the disease most likely to
be causing this constellation of symptoms?

a. Complete blood count

b. Erythrocyte sedimentation rate

c. Viral culture

d. Clinical judgment

Solution. D) Clinical judgment

Answer. d

278. A 6 month old male presents with failure to thrive , eczema, and a history of recurrent bacterial
infections. On evaluations, a thrombocyte count of 20000/mm3 is noted. The peripheral smear reveals
microthrombocytes. Which of the following is the most likely condition causing these signs and
symptoms?

a. Wiskott- Aldrich syndrome

b. 22 q 11 deletion syndrome

c. Celiac disease

Solution. A) Wiskott- Aldrich syndrome


The Wiskott- Aldrich syndrome, an X- linked disorder, typically presents during the first 6 months
of life with recurrent infections or bleeding episodes. The unusual aspect of the thrombocytopenia
is that the thrombocytes are very small and thus called microthrombocytes. These small platelets
are not common in any other thrombocytopenic disease. The treatment of choice is with HLA-
matched bone marrow transplantation.

Answer. a
279. A 4 year old male presents with 1 day history of abdominal pain and vomiting . He is afebrile and
has no diarrhea. He complains of knee pain bilaterally, and there is some tenderness of the knee joints
but no effusions. Within 24 hours he develops a rash on his legs and buttocks which is petechial and
purpuric, and his platelet count is normal. What is the most likely diagnosis ?

a. Hemolytic uremic syndrome

b. Henoch Schonlein purpura

c. Acute glomerulonephritis

d. Kawasaki disease

Solution. B) Henoch Schonlein purpura

Answer. b

280. 10 The neonatal evaluation for sepsis is complete and the neonate is placed on antibiotics. The
next day there is no evidence of infection and the infant has not improved. Additional studies are
obtained, including a serum ammonia which is 1150 mmol/L (nL 64-107) and a blood pH of 7.36.
Medical therapy is initiated, The urinary orotic acid level returns and is markedly elevated as well.
Which of the following is the most likely diagnosis?

a. Ornithine Transcarbamylase Deficiency

b. Carbamoyl Synthetase Deficiency

c. Methylmalonic Academia

d. Carnitine Palmitoyl Transferase Deficiency

Solution. (a) Ornithine Transcarbamylase Deficiency

Answer. a
281. Which of the following waves on ABR testing is the most sensitive to hearing loss?

a. Wave I

b. Wave III

c. Wave V

d. All of the above

Solution. a) Wave I
Ref: Read the text below
Sol:
NEURODIAGNOSIS WITH ABR
- Wave V is often present even in the presence of significant hearing loss.
- Wave I is the most sensitive to hearing loss; 40 dB of hearing loss can make it difficult to see
wave I at higher frequencies.

Answer. a

282. The genetic transmission of otosclerosis is which of the following?

a. Sporadic

b. Autosomal recessive

c. Autosomal dominant

d. Autosomal dominant with variable penetrance

Solution. (c) Autosomal dominant


Ref: Read the text below
Sol:
Otosclerosis
-It is a disease of bone that is unique to the otic capsule.
-It may cause a conductive hearing loss, a mixed conductive-SNHL, or occasionally a pure SNHL.
-The most common area for stapedial fixation in otosclerosis is the fissula ante fenestram.
-It is an autosomal-dominant hereditary disease with variable penetrance and expression.
-Two-thirds of patients are women

Answer. c
283. A retracted tympanic membrane adherent to the promontory is described as:

a. Tos I

b. Tos IV

c. Sade IV

d. Sade III

Solution. c) Sade IV
Ref: Read the text below
Sol:
Sade classification of retraction of pars tensa
Grade 1 – mild retraction
Grade 2—severe retraction—retracted TM touching incus or stapes
Grade 3—Atelectatic TM—TM touching promontory but moves on seigelisation
Grade 4---Adhesive TM--TM touching promontory, does not move on seigelisationos classification
of pars flaccida
Tos classification of pars flaccida
Stage I: Pars flaccida is dimpled and is more retracted than normal. It is not adherent to the
malleus.
Stage II: In this stage the retraction pocket is adherent to the handle of malleus. The full extent of
the retraction pocket can be clearly seen.
Stage III: In this stage part of the retraction pocket may be hidden. There may also be associated
erosion of the outer attic wall (scutum).
Stage IV: In this stage there is definite severe erosion of the outer attic wall. The extent of the
retraction pocket cannot be clearly seen as most of it are hidden from the view.

Answer. c

284. All of the following are true about bone anchored hearing aid except

a. It has surgically implantable titanium fixture

b. it transmits sound directly to cochlea through the bone conduction using the principle of
osseointegration

c. It is indicated in patients with chronic discharging ear

d. It is not useful in unilateral severe sensorineural hearing loss

Solution. d) It is not useful in unilateral severe sensorineural hearing loss


Ref: Read the text below
Sol:
- BAHA- bone anchored hearing aid will be given to hearing impaired individuals where
conventional hearing aids can not be given like in bilateral Anotia, EAC atresia, discharging ear,
EAC stenosis.
- Its a specialized device which is surgically fitted partly on to mastoid bone. it transmits sound
directly to cochlea through the bone conduction using the principle of osseointegration

Answer. d
285. All are true about benign paroxysmal positional vertigo except

a. Semont man oeuvre is done to treat this condition

b. Otoconia most commonly migrate to lateral semicircular canal to cause this entity

c. There is no hearing loss in this condition

d. Calcium carbonate are in involved

Solution. b) Otoconia most commonly migrate to lateral semicircular canal to cause this entit
Ref: Read the text below
Sol:
BPPV
Otoliths or otoconia are calcium carbonate crystals lying embedded in the gelatinous layer of
macula. Macula is the sensory end organ of utricle and saccule whose function is linear
acceleration.
BPPV
- Displaced otoconia from macula is the aetiology
- Most commonly otoliths come to lie in posterior semicircular canal
- Dix hallpike’s test is diagnostic
- Epley’s maneuver Or Semont’s maneuver is therapeutic which is particle repositioning maneuver

Answer. b

286. A patient of head injury has bluish discoloration over mastoid area, bleeding from EAC and
complaining of hearing loss with normal facial nerve function. After three days, patient starts having
facial weakness. What is the possible diagnosis?

a. Longitudinal fracture of temporal bone

b. Transverse fracture of temporal bone

c. Traumatic perilymphatic fistula

d. Traumatic fracture of parietal bone

Solution. (a) Longitudinal fracture of temporal bone


Ref: Read the text below
Sol:
- This is a case of longitudinal fracture of temporal bone. The ear bleeding is a feature of
longitudinal fracture. This fracture is due to fall on the side. Battle sign is ecchymosis around the
mastoid tip area which is seen in temporal bone fractures. This fracture causes ossicular
dislocation and hence conductive hearing loss. Transverse fractures lead to cochlear injury and
hence cause SNHL. CSF otorrhoea is more common with longitudinal fractures
- In temporal bone fractures, the facial palsy is either of immediate (Transverse fractures) or
delayed onset ( Longitudinal fractures)
- Immediate onset need immediate surgery which is facial nerve decompression and if need be
end to end anastomosis or nerve grafting
- Delayed onset should be managed with 3 weeks of steroid therapy as the cause is oedema in this
case. If there is no recovery after 3 weeks of steroid therapy, electrophysiological nerve testing
should be done ( Electroneuronography). If there is significant facial nerve degeneration, patient
should be taken for surgery (facial nerve decompression) should be done.

Answer. a
287. In a patient of acute epiglottitis, all of the following are part of management except

a. Intubation to secure the airway

b. Steroids to decrease edema

c. Antibiotics to control infection

d. Repeat direct laryngoscopy every hour to assess laryngeal edema

Solution. d) Repeat direct laryngoscopy every hour to assess laryngeal edema


Ref: Read the text below
Sol:
- In Acute Epiglottitis, the airway establishment by intubation or tracheostomy is the first
treatment to be given.
- But repeated direct laryngoscopy should be avoided as this can aggravate the edema of larynx,

Answer. d
288. A 6-year-old boy is brought to hospital at 8 PM because he told his parents that he swallowed
something but did not say what it is. He is not in distress and is swallowing his saliva. His chest x-ray is
shown below. There is currently a laparotomy in the emergency operating room that will take another
4 hours to complete. What is next step in management?

a. Operate after 4 hrs

b. Second operating room needs to be opened and the operation performed regardless of his
fasting time.

c. Conservative management

d. Advise HRCT to locate actual position of the foreign body

Solution. b) Second operating room needs to be opened and the operation performed regardless
of his fasting time.
Ref: Read the text below
Sol:
- In this case it is unclear what the object is.
- If you look closely, there is a ‘ring’ at the edge of the object, which raises the suspicion of a
button battery.
- If this is suspected, it must be treated as a life-threatening case and removed as soon as possible
as he is at high risk of perforation and its subsequent sequelae.
- A second operating room needs to be opened and the operation performed regardless of his
fasting time

Answer. b
289. Study the given tracheostomy tube, it is being used for:

a. Preventing Aspiration

b. Voice

c. Reducing pressure necrosis

d. None of the above

Solution. (b) Voice


Ref: Read the text below
Sol:
- Some tubes have single or multiple fenestrations on the superior curvature of the shaft (see
image below).
- Fenestrations permit airflow, which, in addition to air leaking around the tube, allows the
patient to phonate and cough more effectively. That these tubes allow for patient speech is an
important feature.

Answer. b
290. The surgical demarcation between level III and level IV in the neck is

a. The upper border of the omohyoid muscle.

b. The lower border of the omohyoid muscle.

c. A horizontal line along the inferior border of the cricoid cartilage.

d. The transverse cervical vessels.

Solution. (c) A horizontal line along the inferior border of the cricoid cartilage.
Ref: Read the text below
Sol:
The demarcation between level III and level IV nodes has been given variably as the "cricoid
cartilage or the "bottom of the cricoid arch or the "cricothyroid membrane.

Answer. c
291. A 65 year old patient was diagnosed as rhegmatogenous retinal detachment.
Which of the following is least likely a predisposing feature of it?

a. Lattice degeneration

b. Posterior vitreous detachment

c. Weiss ring

d. Snail track degeneration

Solution. c
Posterior vitreous detachment refers to separation of the vitreous from the neurosensory retina.
PVD occurs due to vitreous gel liquefaction with age
(synchysis) to form fluid-filled cavities and can also induce retinal breaks
Weiss ring is suggestive of complete Posterior vitreous detachment. It is seen by the patient as a
circle or other large solitary lesion. It suggests that traction by PVD is complete, so chances of
retinal detachment is least.
Lattice is area of retinal thinning with variable atrophy of the underlying Neurosensory retina.
Snail track degeneration is characterized by sharply demarcated
bands of tightly packed ‘snowflakes’ that give the peripheral retina
a white frost-like appearance. It is viewed as a precursor to lattice degeneration

Answer. c

292. A 28-year-old has progressive dementia, jaundice and akinetic rigidity in his upper limbs.
On ophthalmic examination, there was cataract seen. The doctor also observed brownish deposition on
cornea.
Which layer in cornea will the deposition will most likely be?

a. Epithelium

b. Descement’s membrane

c. Bowman membrane

d. Endothelium

Solution. b
Wilson disease (hepatolenticular degeneration) is a rare condition involving the widespread
abnormal deposition of copper in tissues. It is caused by a deficiency of caeruloplasmin, the major
copper-carrying blood protein. Presentation is with liver disease, basal ganglia dysfunction or
psychiatric disturbances. A Kayser– Fleischer ring is present in 95% of patients with neurological
signs, and consists of a brownish-yellow zone of fine copper dusting in peripheral Descemet
membrane. Anterior capsular ‘sunflower’ cataract is seen in some patients

Answer. b
293. An ophthalmologist wants to check fundus of a myopic patient (-4 Dioptres). If he observes
magnified view of retina 3.2 times,
Which condensing lens is he using?

a. 30 D

b. 28D

c. 20D

d. 14D

Solution. C
Magnification with Fundoscopy (3.2) = Power of eye (64D)/power of condensing lens

Answer. c

294. When the right trochlear nerve nucleus is having lesion, which of the following would not be true?

a. lesion is in midbrain at level of inferior colliculus

b. leads to right superior oblique palsy

c. Left eye will be hypertropia

d. Bielschowsky test is used to see hypertropia is worse on head tilt

Solution. b
Right 4th nerve nucleus supplies left Superior oblique muscle
Left SO palsy will have left hypertropia

Answer. b
295. The finding can be seen in all except

a. Niacin toxicity

b. Diabetic macular edema

c. Branch retinal vein occlusion

d. Best disease

Solution. D
Cystoid macular oedema results from the accumulation of fluid in the outer plexiform and inner
nuclear layers of the retina with the formation of tiny cyst-like cavities.
Seen after Ocular surgery and laser, e.g. phacoemulsification
Retinal vascular disease like Diabetic retinopathy, Vein occlusions
Inflammation, e.g. intermediate uveitis
Drug-induced, e.g. topical prostaglandin derivatives, niacin, miotics

Answer. d
296. All findings can be seen in this figure except

a. Iris coloboma

b. Iridoschisis

c. Polycoria

d. Corectopia

Solution. A
Iridoschisis is atrophy of iris tissue
Corectopia- decentered pupil as in Axenfeld Reiger syndrome
Polycoria – multiple pupil
Iris coloboma is a wedged –shaped defect that occurs as an isolated anomaly or secondary to the
failure of closure of optic fissure. Iris colobomas are typically located inferiorly and may be
associated with colobomas of the uvea.

Answer. a

297. A patient with Marfanoid habitus has subluxation of lens with positive cyanide nitroprusside test
in urine. What could be the diagnosis?

a. Marfan’s syndrome

b. Homocystinuria

c. Stickler syndrome

d. Sulfite oxidase deficiency

Solution. B
Homocystinuria: This is transmitted as an autosomal recessive disease. A deficiency in the enzyme
cystathionine synthetase gives rise to excessive amounts of homocysteine in the urine and
widespread abnormalities characterized by dislocation of the lens and mental retardation.
Homocystine in the urine is detected by the cyanide nitroprusside test. Such patients are poor
operative risks because of the tendency to venous thromboses. Other signs include laxity of joints
and a marfanoid habitus.

Answer. b
298. Associated ocular features in a child with this feature will be all except

a. Shallow anterior chamber

b. Increased corneal diameter

c. High Intraocular pressure

d. Flat lens

Solution. A
Discrete corneal opacities appear as lines with a double contour (Haab striae, due to rupture of
Descemet’s membrane) in congenital glaucoma
Anterior chamber is deep

Answer. a
299. An ophthalmology resident was asked to observe pupil reactions in dark, light and with near
object. If he sees the reactions like this, what is the likely pathology

a. Horner syndrome

b. Homes-Adie pupil

c. Argyll-Robertson Pupil

d. Marcus Gunn pupil

Solution. B
Lesion of the ciliary ganglion: abolition of the light reflex with retention of the near reflex,
sometimes with tonic contraction (Adie pupil).
Here there is right side Adie pupil.

Answer. b
300. From what distance can a 6/24 patient can see the 6/24 on Snellen chart:

a. 60 m

b. 36 m

c. 24 m

d. 6m

Solution. D
A 6/6 patient can see
6/6 letter from 6m
6/60 (topmost letter) from 60m
6/24 from 24 m and so on
6/24 patient can see the 6/24 letter from 6 m.

Answer. d

Test Answer

1 (a) 2 (d) 3 (b) 4 (b) 5 (a) 6 (c) 7 (c) 8 (c)


9 (b) 10 (a) 11 (b) 12 (b) 13 (c) 14 (c) 15 (b) 16 (c)
17 (a) 18 (d) 19 (d) 20 (a) 21 (a) 22 (a) 23 (a) 24 (d)
25 (c) 26 (b) 27 (d) 28 (d) 29 (d) 30 (c) 31 (c) 32 (b)
33 (b) 34 (a) 35 (c) 36 (c) 37 (c) 38 (b) 39 (c) 40 (b)
41 (c) 42 (b) 43 (d) 44 (d) 45 (d) 46 (a) 47 (d) 48 (c)
49 (c) 50 (d) 51 (c) 52 (a) 53 (c) 54 (a) 55 (b) 56 (d)
57 (d) 58 (c) 59 (b) 60 (b) 61 (c) 62 (b) 63 (d) 64 (c)
65 (b) 66 (d) 67 (c) 68 (b) 69 (d) 70 (b) 71 (a) 72 (b)
73 (c) 74 (c) 75 (a) 76 (b) 77 (c) 78 (d) 79 (d) 80 (c)
81 (c) 82 (d) 83 (b) 84 (a) 85 (b) 86 (a) 87 (a) 88 (a)
89 (b) 90 (b) 91 (d) 92 (b) 93 (d) 94 (c) 95 (b) 96 (a)
97 (a) 98 (a) 99 (d) 100 (b) 101 (d) 102 (b) 103 (a) 104 (d)
105 (d) 106 (b) 107 (a) 108 (b) 109 (d) 110 (a) 111 (a) 112 (b)
113 (c) 114 (d) 115 (a) 116 (a) 117 (d) 118 (b) 119 (b) 120 (a)
121 (c) 122 (a) 123 (c) 124 (c) 125 (c) 126 (c) 127 (b) 128 (a)
129 (b) 130 (b) 131 (a) 132 (c) 133 (b) 134 (c) 135 (c) 136 (a)
137 (b) 138 (c) 139 (b) 140 (d) 141 (d) 142 (a) 143 (c) 144 (a)
145 (d) 146 (d) 147 (d) 148 (b) 149 (b) 150 (a) 151 (d) 152 (a)
153 (d) 154 (b) 155 (c) 156 (d) 157 (b) 158 (c) 159 (a) 160 (d)
161 (d) 162 (b) 163 (a) 164 (b) 165 (b) 166 (d) 167 (c) 168 (c)
169 (b) 170 (a) 171 (c) 172 (c) 173 (c) 174 (c) 175 (b) 176 (b)
177 (b) 178 (c) 179 (c) 180 (c) 181 (c) 182 (d) 183 (b) 184 (b)
185 (a) 186 (c) 187 (c) 188 (c) 189 (c) 190 (c) 191 (b) 192 (b)
193 (b) 194 (b) 195 (c) 196 (c) 197 (b) 198 (b) 199 (d) 200 (b)
201 (d) 202 (b) 203 (c) 204 (b) 205 (d) 206 (b) 207 (c) 208 (b)
209 (b) 210 (d) 211 (c) 212 (b) 213 (c) 214 (c) 215 (c) 216 (b)
217 (b) 218 (c) 219 (c) 220 (a) 221 (b) 222 (a) 223 (a) 224 (a)
225 (d) 226 (c) 227 (c) 228 (d) 229 (c) 230 (d) 231 (d) 232 (d)
233 (b) 234 (c) 235 (c) 236 (b) 237 (c) 238 (d) 239 (c) 240 (c)
241 (c) 242 (c) 243 (c) 244 (a) 245 (c) 246 (d) 247 (d) 248 (b)
249 (a) 250 (d) 251 (b) 252 (c) 253 (b) 254 (c) 255 (b) 256 (d)
257 (d) 258 (c) 259 (d) 260 (b) 261 (c) 262 (a) 263 (c) 264 (c)
265 (b) 266 (d) 267 (c) 268 (d) 269 (b) 270 (b) 271 (a) 272 (b)
273 (a) 274 (d) 275 (d) 276 (d) 277 (d) 278 (a) 279 (b) 280 (a)
281 (a) 282 (c) 283 (c) 284 (d) 285 (b) 286 (a) 287 (d) 288 (b)
289 (b) 290 (c) 291 (c) 292 (b) 293 (c) 294 (b) 295 (d) 296 (a)
297 (b) 298 (a) 299 (b) 300 (d)

You might also like